You are on page 1of 176
MODULAR SYSTEM Analytic Analysis of LINES and CIRCLES Muhammer Toskiran Cnet Kilg Zambak Zambak a Copyright © Surat Bosim Reklomalk ve Egii Araglan Son Tic. AS. All rightsreseved No port of tis book may he reproduced, stored ino retrieval system or transmitted in any form with- out the prior written permission of the publisher. Digital Assembly Tombak Typesetting & Design Page Design ‘Somil Keskinodlu Serdar Cam Proofreader Toe Barnet Poblsher Tombok Basim Yoyin Egitim ve Turiam Isletmelei Sanayi Ticaret AS. Printed by GoBlayon A. Soom ol ier No.7 aztec xm, Febcoury 2008 Tel: +90-0-232-252 22 85, +90-0-232-522-20-96-97 ISBN: 975-266-026-6 Printed in Turkey DISTRIBUTION ‘TAMBAK YAYINLARI Bulgurlu Noh, Hominne Cesmesi Sok No. 20 34696 Uskiidar / Istanbul Tal: +90-216 522 0900 (pbx) Fox: + 90-216 443 98 39 hitp://book zombak com PREFACE To the Teacher, Analytic Analysis of Lines and Circles 1s designed to provide students with the analytic geometry background needed for further collegelevel geometry courses. Analytic geometry can be defined as algebraic analysis applied to geometrical concepts and figures, or the use of geometrical ‘concepts and figures ro illustrate algebraic forms, ‘Analytic geometry has many applications in different branches of science and makes it easier to solve a wide vanity of problems. The goal of this texts to help students develop the skills necessary for solvmg snalyne geomeny problems, and then help students apply these skills. By the fend of the book, students will have a good understanding of the analytic approach to solving problems. In addition, ‘ve have provided many systemauc explanations throughout the text that will help instructors to reach the goals that they have set for their studemts, As always, we have taken parricular care to create a book that students can read, understand, and enjoy, and that wil help students gain confidence in their ability to use analytic geomem To the Student, Check Yourself 13 1, What isthe equation ofthe lin joining the 2. Find the equation ofthe Kine passing th '. Find the value of the above line as Answers Le x48 Qu geeay ‘This book consists of two chapters, which cover analytical analysis of nes and circles respectively, Each chapter begins with basic definitions, theorems, and explanations which are necessary for understanding the subsequent chapter material. In addition, each chapter is divided into subsections so that students can follow the material easily. Every subsection includes self-test Check Yourself problem sections followed by basic ‘examples ilustrating the relevant definiuon, theorem, rule, or property: Teachers should encourage their students to solve Check Yourself problems themselves because these problems are fundemental to understanding and Iearning the related subjects or sections. The answers to most Check Yourself problems ae gyen directly after the problems, so that students have immediate feedback on their progress. Answers to some Check Yourself problems are not meluded m the answer key, as they are basic problems which are covered in detail in the preceding text or examples. Gwing answers to such problems would effecnvely make the problems redundant, so we have chosen to omit them, and leave students to find the basic answers themselves. ii ers At the end of every section there are exercises categonzed EXERCISES 1,1 . according {0 the structure and subject matter of the Ae Abate nnyale af neta section. Exercises are graded in order, from easy (at the areca beginning) to difficult (at the end). Exercises which involve more ability and effort are denoted by one or two stars. In addition, exercises which deal with more than ‘one subject are included in a separate bank of mixed problems at the end of the Section, This organization allows the mstructor to deal with only part of a section Af necessary and to easily determine which exercises are appropriate (0 assign. ng pints in the coontinate pi eee) © HO) £ Fo) Every chapter ends with three important sections, ‘The Chapter Summary isa list of important concepts and | (RAMANA LLNS 6 formulas covered in the chapter that students can use | + ‘hereisa one-to-one cnrespondencehermen th easily (0 get direct information whenever needed, Sas tb reprsenad by two components the alc Concept Check, 01 = 5 A seetion contains cninatey AC 9). Taran aetonesin questions about the main concepts of the subjects covered, 2) How cn a point be presented the coordinate eSPeCially about the definitions, theorems or derived 3. Define the eacept of te. Find exaries om | formulas. Finally, a Chapter Review Test section consists of three tests, each with sixteen carefully-selected problems, The first test covers prumitive and basic problems. The second 1. Whats the lent of dhe mesian pss dro tho vores ofa uae ABC wih vores, and Uurd tests mclude more complex problems. These | Bi-1,2), ai e«07 tests help students assess their ability in understanding | 36 7B) the coverage of the chapter. ‘The answers to the exercises and the tests are given at the end of the book so that students can compare their solution wath the correct answer. G.BUNCH OF LINES (OPTIONAL) Each chapter also mcludes some subjects which are denoted as optional, These subjects complement the topic and give some additional information, However, —— completion of optional sections 1s left to the discretion of, the teacher, who can take mto account regional curriculum requitements, Acknowledgements We would fike to thank the many people who helped 8s prepare the text and supplement package. Their encouragement, cohstructive ‘comments, and suggesti6ins were invaluable fo us. First of all we woulgif like! to sincérely thank Mr Ramazan Sahin forgiving us his research and studies, which formed the: essé base for our project. We would thank Mr. Orhan Keskin, Mr Ali Gavelar, Me cand Mr Mustafa Kirikgt who encourag: i us soadiich while we were developi ‘ing, ‘isi, bald Publishing who worked withbs, especially to Samil K@sKinogly and Sear Cam for their precious efforfSin typing and illustrating the material, and nie Bho supp liedinnmelud ble cont Bulone re. Seabe iiGinks in addition #Blell the people: at during 1 riod of the preparation of the project, Final febumthagks ga felibur families tend wonderful childrentwhe gayé us so much suppor by understanding and helping us. The Authors INTRODUCTION ‘CHAPTER 1: ANALYTIC ANALYSIS OF LINES 1, THE COORDINATE PLANE A. ANALYTIC ANALYSIS OF POINTS ....2 B, ANALYTIC ANALYSIS OF TRIANGLES . EXERCISES 1.1 - ANALYTIC ANALYSIS OF UNES A. TRIGONOMETRIC RATIOS. OF ANGLES «2.0 ..00005 B SLOPE OF ALINE . EQUATION OF ALINE é D. FINDING THE SLOPE OF A LINE WITH A GIVEN EQUATION RELATIVE POSITION OF TWO LINES ... FIRST DEGREE INEQUALITIES IN TWO UNKNOWNS . 20.0... .55 G. BUNCH OF LINES (OPTIONAL) .. . .57 EXERCISES 1.2... 60 FURTHER APPLICATIONS ‘A. ANGLES BETWEEN TWO LINES... .67 B. DISTANCE FROM A POINT TO ALINE .. C. DISTANCE BETWEEN TWO. PARALLEL LINES . . D. EQUATIONS OF ANGLE BISECTORS . EXERCISES 1.3. SYMMETRY eas 2.20 rier eee 25 = FE A. SYMMETRY OF A POINT ........+.78 83 BR SYMMETRY OF A LINE (OPTIONAL) EXERCISES 1.40... CHAPTER SUMMAI CONCEPT CHECK CHAPTER 2: ANALYTIC ANALYSIS OF CIRCLES 1. EQUATION OF A GRCLE ‘A. STANDARD EQUATION OF AGIRGUED crermese canes seh 8. GENERAL EQUATION OF ACIRCLE .... + 106 EXERCISES 2.1 5.0.2... 00600004.-113 2. POSITIONS OF WINES AND CIRCLES A. RELATIVE POSITION OF A LINE AND A CIRCLE “5 B. EQUATIONS OF NORMAL LINES... ong C. RELATIVE POSITION OF TWO CIRCLES 126 EXERCISES 2.2 130 POWER OF A POINT, RADICAL AXIS, AND RADICAL CENTER ‘A. POWER OF A POINT 132 B. RADICAL AXIS OF TWO CIRCLES . .136 C. RADICAL CENTER OF THREE GRCIES po qacsees means aD ENXERGISES 23) cscs scse ve cacess oss AT 4. FURTHER APPLICATIONS ON CIRCLES (OPTIONAL) A. PARAMETRIC EQUATIONS OF A CIRCLE 43 B. EQUATION OF A SEMI-CIRCLE “145 C. INEQUALITIES INVOLVING CIRCLES . .147 1D. FAMILY OF CIRCLES 2... 0.020004. 0149 puzzi EXERCISES 2. CHAPTER SUMMARY CONCEPT CHECK . ANSWERS TO EXERCISES . ANSWERS TO TESTS Glossary... INTRODUCTION René Descartes (1596 - 1650), also known as Cartestus, worked as a philosopher and mathematician. While most notable for his groundbreaking work in philosophy, he achieved wide fame as the inventor of the Cartesian coordinate system, which influenced the development of modern calculus. Descartes, sometimes called the founder of modern philosophy and the Father of Modem Mathematics, ranks as one of the most important and influential thinkers in human history. Descartes invented what we now call Cartesian coordinates, or the system by which we can graph mathematical functions in two- or three-dimensional space. Mathematicians consider Descartes of the utmost importance for his discovery of analytic geometry, Up to Descartes’ time, geometry, which dealt with lines and shapes, and algebra, which dealt with numbers, appeared as completely different subsets of mathematics. Descartes showed how to translate (almost) all problems in geometry into problems in algebra, by regarding them as questions asking for the length of a line segment, and using a coordinate system to describe the problem. It is interesting to consider how Descartes came to develop Cartesian coordinates Descartes was lving on his bed watching a fly Slowly it came to him that he could describe the fly's position at any instant by just three numbers. Those three numbers were along the planes of the floor and two adjacent walls, what we now call the x, y, 2 coordinate system. Most cities today are laid out in Cartesian fashion, so we give directions as x, y coordinates easily and without realizing it. Descartes’ theory provided the basis for the calculus of Newton and Leibniz, and thus for much of modern mathematics. This appears even more astounding when one keeps in mind that the work wasjust meant as an example to his Discourse on the Method to Rightly Conduct the Reason and Search for the Truth in Sciences, known better under the shortened tide Discours de la méthode. In part two, he introduces the new idea of SAN. ~ =< specifying the position of a point or object on a surface, using two intersecting axes as measuring guides. Another book by Descartes, La Géométrie, was published in 1637. The book was influential in developing the Cartesian coordinate system and specifically discussed the representation of points of a plane, via real numbers, and the representation of curves, via ‘equations, The modem Cartesian coordimate system in two dimensions (also called a rectangular coordinate system) is commonly defined by two axes, at right angles to each other, forming a plane (an xy-plane). The horizontal axis is labeled x, and the vertical axis is labeled y. In a three-dimensional coordinate system, another axis, normally labeled z, is added, providing a sense of a third dimension of space measurement. The axes are commonly defined as mutually orthogonal to each other (each at a right angle to the other), Early systems, however, allowed oblique axes, that is, axes that did not meet at right angles. All the points {in a Cartesian coordinate system taken together form the so-called Cartesian plane. t } ( ‘iD THE COORDINATE PLANE } A. ANALYTIC ANALYSIS OF POINTS 1. Basic Concepts coordinate plane ‘Two number lmes which are perpendicular to each other at the origin (pomt zero) form the Cartesian coordinate system (also called the rectangular coordinate system), and the plane used for this is called the coordinate plane. Ga In the coordinate plane, the horizontal ama TT line is called the 2-s and te vertical . line is called the y-axis, ‘ tureetion| 5 arant 4 quadrant These axes divide coordinate plane into four quadrants, (regions), as shown in the figure The pomt O(0, 0) 1s called the orign of the coordinate plane. In the coordinate plane, we can specify the exact location of any point by giving its.x- and y-coordinate. Analytic Analysis of Lines and Cirles Solution The Coordinate Plane Let us take a point P in any quadrant of the coordinate plane and draw perpendicular line segments from P to the x- and y-axis which meet the coordinate axes at a and b, as shown am the diagram. The real number a on the x-axis is called the coordinate or abscissa of point P. The real number b on the y-axisis called the y-coordimate or ordmate of point P. The ordered pair (a, b) 18 called the coordinates of the point P and 1s denoted by P(a, b). Plot the points AC, -2), BC3, 4), C2, 1, DG, 2), and E(-L,4) am the coordinate plane, Now ook at the diagram. Notice that the ordmate of every pomt on the x-axis is 2er0, the ae abseassa of every pomt on the y-axis 15 zer0, and ne the coordinates of the origin are (0, 0). x 02 Q Poe me poms KA, 0, LO, 9). MCS, 0), and N(O,-2) inthe coordinate plane, Gwen a point P(a, b) an the coordmate plane, 1, @>0,b > Owhen P ism the first quadrant 2. @ <0,b> O when P ism the second quadrant 3. a <0, b <0 when P ism the third quadrant 4, @ > 0, b < 0 when P ism the fourth quadrant. (ETLTNGY 3 wehe pome fa, ab) wim the second quadrant ofthe coordinate plane, in which quadrant as the pomt Ba’, *) pomt Bia, £5? Solution If (, y)isim the second quadrant, then x < 0, y > 0. A(a, @-b) 183m the second quadrant, so @ < O and a:b > 0. Therefore, b < 0 also For the point B(@",*), a >0 and > 0 Hence, B as in the first quadrant, (EOL 4 the poms Afa, a 2b) and BC, b) ate coincident, Find their coordinates and state which = quadrant they are sn. Solution A(@, a 2b) = BC3, b) a= -Sanda-2 . $0 b = =1 and so A(-3, ~1) 1s m the Uurd quadrant. Analytic Analyee of Lines and Cites SETS the sides of a rectangle ae parallel to the coordinate Solu The Coordinate Plane axes, and the length of the zectangle 15 four times ats, width. If the perimeter P of the rectangle 1s 50 umsts, and D(8, 2) 1s its vertex, find the coordinates of the points A, B,C. The sides of the rectangle are parallel to the x- and 1-235, $0 they are perpendicular to the axes at pomts E.F,G.H. Here, AD = 4. DC and P = 2- (AD + DC) = 2- (4-DC + DC) = 10- DC = 50 ums. So DC = 5 units. DC = DF + FC and DF = 2.umtts, so FC = DC - DF = 3- = 3.unuts. AD = 4. DC = 4-5 = 20 ums, Sumilazly, AG = AD — GD = 20- and so the coordinates are A(-12, 2), B(-12, ~3), and C(8, -) = 12 ums, Check Yourself 1 1, Plot the pomts AC3, 0), ae, 2), C0, > D2, -3. and E(-9,-4) in the coordinate plane. 2, Find the cooramates of the pomts which are 5 units from the x-axis and 2 units from the y-axis 8, Draw the uiangle with veruees AL, 7), BC2, 8), and C2, -2) am the coordinate plane. 4, Ifthe point Mn, n) is mn the third quadrant of the coordinate plane, find the quadrant containing the pout Nn, m), 5 If the pomt P(m + 1, m — 6) isan the fourth quadrant of the coordinate plane, find the ‘possible mteger values of m. Answers 5. (0, 1,2, 3, 4. 5} 2. Distance Between Two Points Let Us Use ay yaa, AMM My yp Yoyo UO denote the abscissas and the ordinates of respective pomts in the coordinate plane distance between two pots The distance between two points A(x, ¥,) and Bt, ¥,) 18 Proof In the figure, ABC isa right tangle. AC =x,-x, BC=H-¥, By the Pythagorean theorem, AB’ = AC’ + BCT ABE = (xe) + 9)" and so AB = ¥@s—&)) F Gn wy oF = VaR nye G Hind the distance between A(3, 0) and BC2, -3) PF ny = 2-3 +0» = fe 3) Solul 34 uns, (MEZOELETY 7 show thar ABC withthe vertices AC-2, 2), B(, 8), and C(A,-1) isan isosceles triangle Solution Let us find the length of the sides of ABC. AB= +2) + G2)" = 79-30 Ac = (GaP FLO = TT = Be = fa FCF = PTI AC = BC, so two sides of the mangle have the same length. ‘Therefore, ABC is isosceles. & 40, 2,86,., and C2, 9 ar gven. IFA atthe sume distance fom the pons Band C, finda Solu ‘We are gven AB = AC. By the theorem for the distance between two points, faray + 9-6at a +4= gp 4404441 10a =8 4 = peti ty fie an Cdr Find tne ordmate ofthe pont on the y-ax which s equidistant to the porns AC-4, 0) and Solutio Proof The Coordinate Plane BOQ, 9). The pomt is on the y-axis, so Its x-coordinate 1s 0, Let us call the pomt P(0, it). Now, from the diagram, PA= PB (OTE = Pay Cay ke = 9 +e)! 16 +H = 81 +R 10k +29 10k = 90 kt = 9, Therefore, the point 1s P(O, 9). Check Yourself 2 |. Fimd the distance between the pomts A, ~1) and B(-2, 2) 2. Find the lengths of the sides of the tangle MNP with vertices at the pomts M(-1, 3), N(2,-3), and PS, 1) 8. The pomts K@, 1) and LC6, a) are gwen. If KL = 10 cm, find the possible values of a 4. Aisa point on the y-axis with ordiate 5 and B 1s the pout (-3, 1). Calculate AB. 5. Find the pomt on the y-axis which 1s equidistant to the pots AC-3, 0) and BC, -1). Answers 1.5 2. ¥87, 2/10, 465 3a E57) 45 9.0,-4) 3. Midpoint of a Line Segment amidpomt of @ bne segment Let the pomts A(x, y,) and B(&, ¥,) be the endpomts ofa le segment AB, and let C(x, 1) be the midpomt of AB. Then, ae a and y= WA Ye 2 Let us take pomt C on AB such that AC = CB From the figure, ACAK = ABCD. So AK = CD and CK = BD. Now, %5-x Xo and Yo Wy = Yep a ee atm uti x= S28 and y= Mitte 2 4 2 te wt S0CO, Ho) 2° 3 ESE 10 12. -2) ana 86-5, 4) ave given. Find the coordinates of the midpoint of AB. ate -1-5 wty, 244 2 2 Solution x, 2 2 So C(-3, 1) 1s the midpoint of AB. 3), and C(-2, =1), draw AABC m a (EEQOIET TT oven tne mangle ABC with vertices A@, 3), BQ. coordinate plane and find the lengths of the medians. Let D, £, and F be midpomts of the sides of ABC, and let V,, V,, and V, be the medians as shown in the figure, ‘The coordinates of D are 40,2) (0-0 G42) The coordinates of E are 0-2 a 3- -1 and y, BE = JQ+1°+C3-=6 ums, and (QUE 12 A taangte AB with vertices AC2, -2), BU, 8), and C(6, 2)38 ven, Ifthe pountsD and E are Imidpomts of AB and AC respective show that BD = 5 Solution Furst, let us find the coordinates of D(a, b) and E(¢, d). Pomts D(a, b) and E(c, d) are the midpoints of AB and AC, so their coordinates are = prt ty fie a Cr Now, let us find the length of ED and BC by using the distance formula, and then compare their lengths: fo+ 5 +0-9° Let the points A(x, 9), B(ts, 22), Cts, y.), and D(a) Devan Con be vertices of a parallelogram ABCD, and let P(t 1,) be the mtersection pomt of the diagonals. Since P(x, 1,) 18 the midpomt of the diagonals, a +x na andg Bt so ytysut hao on atu +H = A md y= 50 Wtt= tet Me ‘Asa result, for any parallelogram ABCD with given vertices the following rules are vali xy tx =m Faand y, Y,= Ye tH (EEE 13 ALM sa paraticlogram with vertices K@, a), LC, 4), M@, 3), and N@, 2). Fmd a ~b. ‘The midpoint of KM is also the midpoint of NL, so 24+b=143 and a43=442 b=2 a=3 Therefore, a -b =3-2= 1. The Coordinate Plane Check Yourself 3 1, A(@ + 1,4~2h) and BG ~a, 2h—3) are gyen. Find the coordinates of the undpount of AB 2. A tmiangle AABC with vertices A(2, 6), B(-2, 3), and C(4, ~1) 1s given, Find the length of the median passing through A. 3. The points AC2, 3), BG, -2), C(x, y), and DCI, 8) are the vertices of a parallelogram ABCD. Find the coordinates of C Answers 1 1@5 247 344) @> aa 4, Coordinates of a Point Dividing a Line Segment in a Given Ratio a. Coordinates of a point dividing a line segment internally Let AB be a line segment and C be a point on AB. Pot C dwides AB internally and the rau of the Length of CA to the length of CB 1s CA. cB Ifthe ratio F ts gven, we can find the coordinates of C using the following theorem. Let A(y,, »,) and B(x, y,) be the endpoints of AB, and let C(x, y,) be a pomt on the ime seg- ment AB such that C4 f, (ke R*), Them the coordinates of Care yee Tk Proof In the figure AAKC ~ ACHB ana AK — RC _ AC ca” HB cB x, . W.-Y, = ReYs— Rey, nthxsttn ly yay enyey +h) = 2,4 hex, yt =a, + heny athe, “TER y= the Tek Avalytic Analysis of Lines and Citler 14 Me poms AC, 8) and B(@, -11) are given. Find the coordinates of the point C which divides AB internally the rao A ca cA +hex, _-343-9_ 24 is olution <4=3andby the theorem, x, Solution ZF am they 5 Te So the pamt is (6, -7) Practical Way: cA C(x 4) divides AB internally; We know CA (ew) ae s0 CA = 3CB. So our Ime is divided into four equal parts. Let CB = a, then CA = 3a. poe ae Ber oe C640 8@,-11) To nd x,: AB = 4a and the abscissa changes 9 ~ (-3) = 12 unus. 12 unis ‘Therefore, since AC = a, the absessa vl change 12.3 =9 ums nea Sox,=-34+9=6 k= 9 units ‘To fd y,: AB = 4a and the ordinate changes -11 ~5 = -16 units. Therefore, since AC = 3a, 6 the ordinate will change —2°.3=~12 units. So y : we > fgteum a % ums So the paint is (6, ~7). amuts k= 12 umts 15 The tine segment with tne enaponts M(-2, 3) and NG, -1) 1s given. Find the coordinates of the pomt R which divides MN internally an the ratio MR. _ 2 MIN Solution REMN MR _2 MR _2) > EN rv ar -as2a ; The coordinates of pomt R are x, =__3 = 3 e as 5 a © soune poms net, Note The coordmates x, and x, 1, and tin the formula are specified according to the given ratio mt ‘which the numerator is the distance of the first pomt (x, y,) from the dividing point, and the denominator is the distance of the second pont (x 3) ftom the dividing pomt. Jn the example 15, ‘Mis taken as the first pomt, Nis taken as the second pomt where fis the dividing pomt The Coordinate Pane = Practical Way: ‘We can use a sunlay technique to Example 14. RG, v9) divides MN internat. ME 2, 59 tet us MNS write MR = 2a and MN To find xy: From MN = 9a and the abscissa changes 1 ~ (2) = 3 untts. Therefore, sinee MR = 2a, the 6 oa. abseassa wall change 3-2 = © umnts, 60x, =-2+ 5 ‘To find y,: MN = Sa and the ordinate changes -1 - 3 the ordinate wall chang Let D, A, and B be collinear points such that D hes outside the line segment AB, as shown in the diagram. ‘The ratio of the distance of D to the pois A and B, DA respectively is. 2A = p, DB Ifthe value of fas given, we can find the coordinates of D using the following theorem. Let AG, y,). BG,, y.), and D(x,, y,) be collinear pots such that D ¢ AB then DA €:R*), and the coordinates of D are given by =k es fy ® “Yo Look at the figure, App —apitD ana AP - PP _ AD _ BH HD BD Weneed APB oy ang PMB BH HD u,-. Analytic Analyee of Lines and Cites ES 16 Solution ES 17 The Coordinate Plane Therefore, ak y= yk YAW =Y,-U-Us kU y= BEw Lk ‘The Line segment MN with endpomts M(-2, 3) and N(4, 5) 1s gwen. Find the coordmates of the pom P which divides MN externally inthe rano T= 5 So the coordinates of point P are (16, 9). Practical Way: For x,: The change is 4~ (-2) = 6 units from Mon. 6-3 = 18 unm, sox, =-2 + 18 = 16. For y,: The change is 5-3 = 2 ums from M to N. PM _3 z Paya) 2.3 = 6 umits, soy, = 3 + 6 = 9 and so P(16, 9) is the pomt In ABC, AK = KB, KS = SC, BU, 2) and (4, -A) are given. T is a point such that T © BC and S © AT, as shown in the digram, Find the coordinates of T. a3) T oct.) =— Solution AK BT CS In sane tus we the menaleus theorem, which savs +o" Then 4.37 and 27, 270 1 Te ltk +2 x,=3, and yt hey, 242-4) Tk 12 ‘Therefore, the coordinates of T are (3, -2) Check Yourself 4 1, The bine segment AB has endpoints ACL, 5) and B(4, 2). C sa pomt on the hne segment AB such that “C1. pina the coordmates of € cB 2 A point T divides L4 externally m the abo 7H =. The coordinates ofthe endpoints of LM are L(-1, -9) and M(O, 3). Find the coordinates of the pomt 7. {The pn DQ, aves the ne segment AB external the rao 2 coma of ne 1,3) he cota ofthe pon 4 In the figure, KS = 2 - ST, and pots K-3, 0) and S(0, 2) are given, Find the distance of the point T from the ongin. Answers 39. 3 42 5 35 14 7 456 Avalytic Analysis of Lines and Citler B. ANALYTIC ANALYSIS OF TRIANGLES En 18 Solution En 19 Solution The Coordinate Plane 1. Coordinates of the Centroid of a Triangle Let AABC be a triangle wath vertices A(t, ¥,), BOs, 4), C(x, y,), and let D(a, b) be the midpoint of BC. 4X det Us 2 2 Then a= TGC, 4.) 5 the centro (center of mass) of ABC, mtx, on at2- oT Fk 12 ob Wt and y,=%thb_" 2 i+k 12 XS Pu tH, 3 3 Hence, | G(x,,4,) = GC ‘This formula allows us to find the coordinates of the centroid ofa mange. Find the coordinates of the centroid of AABC with vertices A(J, 2), B@, 3), and C(4, 0) MMA te ty by the fomula, ‘The centwoid of AABC 1s omer = 14244 24340 ‘Therefore, the coordinates of G are Gi 1 r Ss 3 ‘The vertices of ABC are A(-a, -b), B(b, a), and C(2b, 2b), and its centroid is G(1, 9). Find 3 the coordinates of the vertex A. wee 5 IE HO, anase aban 0) 3 3 3 pte 4 Bobet is gots, antto ba=8 0) By solving the simultaneous equations (1) and (2), b= 3, and so. AC3, -2). Check Yourself 5 1. Find the coordinates of the centroid of AABC with vertices A(I, 7), B(-2, -1), and C@, 0). 2, The centroid of ABC with vertices A(-1, 4), B(O, ~3), and C(m, n) 18 GC, 2). Find m +n ' 3 A AOB in the figure isan equilateral triangle. Find the coordinates of the centrosd at aa Answers 2 LGD 29 3.6.28) 2. Area of a Triangle with Known Vertices en of w triangle with: known-vernces Let A(t )), BOs, 40), and CC%,, 4) be the coordinates iG AG, 4 of the veruces of ABC. Then the area of ABC is 1 |A(AABC) = =] xy(ys —4,) + 26-4.) 4 (0, —¥2)] Coe 90 In the figure, A(ABC) = A(ABDE) + A(AEFC) ~ A(BDFC). > Notice that all the polygons ABDE, AEFC, Of<—«oh and BDFC are trapezoids, By the formula for the area ofa trapezoid, we can wnite dey Ys FW) —% z 2 +46 A(AABC) = 1 = FLU DCR) +e) Ce Fay 28) Ca 5) Fy) “Us 9%) ~ Wa ¥3) +e Xe) Os) +s Me 1 A(AABC) = 5 ay(ue —4s)-+ Xap Us) + H(t —4FedT- = Yq ntti nd Cre Ex 20 Solution Pexanrce P| Solution The Coordinate Plane Practical Way: ‘White the coordmates of the vertices in the following form: bitin ‘Now, by using the guidelines which aze positively or negatively ACABO) = "| 2a. denoted, we can wnte the formula - oN 1 DASOWDN, A= SIGs teat a Geom tay ae tH, HL - “and so A= F181) #805 4) 4 C—O Notice that the formula is the same as the formula im the first proof. Find the area of ABC with vertices AC-2, 4), B(3, 2), and C2, -3) Let us wnte A@2, 4) = ACA, 9) BG,2 Ba. 2 Yo) ce, 3 = Cex 99, then AGAABC)= Bx, +(ns 15) #46, BDH “CH -VL =}y-savayeaca-a24-ay[= 4 an-aie4 If the points A(-1, -3), BO, ~2a), and C(3a, a) are three different points on the same line, anda Since A, B, and C are on the same line (collinear), they do not form a tangle, that 1s, AGABC)=0, ACL, -8) = AGL ¥) a3 BO, 20) = Bete 4} = awascy=3.] 2, Pa a3 CGa, a) = Cer, 9.) ‘Therefore, A(AABC) = +] -1-(-2a-a)+0+3a-(-342a)|=0, and so z 3a-9a+6a*=0,s0 a=0 or a=1. Since B@, -2a) and C@a, a) are different pomts, a eannot be zero, so a 2D nd thearea ofa quadrtateral ABCD with vertices AC2,-1), BEB, -2).€(8. 8), and DC-3. 4). Solution Look at the figure. The diagonal BD divides the quadrilateral 7 into two triangles ABD and ABCD. The areas ofthe triangles are Dea AGABD) = 3 -2+(-2-4) 43-4) 3-14 I= 19 1 AMABCD) = 518 -(9-4) +3442) -3-(-2-9) [= 21 So A(ABCD) = A(AABD) + A(ABCD) = 12 +21 = 33. 23 wine heure, BE = ED = DC, AC is perpendicular tothe 7 t-axis, and £(0, 9) and D@, 0) ate given, Find the azea of ABC. cre, 9.) Let us wnte A(x, 9,), BO%, us), and C Gx, 4) BAP 9 5 =2 5 2 BE=ED = = Bt yo #0 2,0) Boy) ED=DC = = 04,3). ‘Since CAs perpendicular to the y-axis, the coordinates of A are (0, 3). Now we have the vertices A(0, 3), B(-2, -6), and C(4, 3), and so the area of AABC is, +[36]=18 units. = ve atte nd Cre Check Yourself 6 |. Find the area of AABC with vertices A(1, 7), BC-1, 3), and CQ, ~4). 2. Find the area of the quadrilateral ABCD with vertices AC2, 1), B(-1, 3), C(4,-1), and DG.4), 8. The pomts AG, 5), BCL, 1), and C2, ky are collinear, Find k. 4 The area of ABC wath veruices AG, 1), B(-1, -2), and C(m, 1) 38 9 squate units, Find the posite ates of m 3. The plats MCL t), NG and PCA, A+ 1 ae om the same ine, Find are Lik gee a Qnty ga 5 ae Seve Yourself by Coorcinates oa Sac Sp gD el a SY caus orm he pa passing thou egres wesy Forex rater uth of Beas inthe world t's Atlante Govan, Is ae automaticaly BAL Arat the number of ciscrpoatdnoes is es expected average for the 250,000 square mls (847 000 sq, km) of emp see Anjuay whatever te truths, fare the coordinates of Wes'30). Fin the late Mes Raises map Write the latitude and longtude of your capital city the place where you were born, and the placer youl. Wc entry ceseefaggeanoftelngld andl ta? The Coordinate Pane O m= tana < 0° m= tant? = 0 vis undelined. Positive slope Neuve slope Ze slope Nosbpe, Avalyic Aral of img lg = 5 Find she sopes of thetines with inchinations 307,43, 60", 1207 and 150" respectively IED 26 EE 27 Solu Solu Solu Inclination (@) Slope (m) 20° tana = 3 40° tan4o*= 1 60° tan60" = 3 120° 8 50° Be 190 = Find the mclnations of the lines with slopes 1 and 0 respecuvely: Sincem =I, tana=-1 = 138° simcem=0, amp =0 Bao Find the slopes of d, and d, in the figure. We have to calculate the tangent values of the angles made by the Lines d, and d, with the positive direction of the x-axis, Fence, m, = tana. me = tan (180° 60°) = = -tan 60° = wa = nd tn often tp = : ng tan (180° 190°) tan 30° Be 3 tte Anais of Line ad Cicer n Proof Arabic Analysis of Lions Let us say 11s the melnation of d, so m = tance. Since a +6 = 180° m = tana. = tan (180-6) tan 2 > Check Yourself 8 1. Find the slope of the line with inclination 135°. 2, Find the inclination of the Line with slope ~1/3. In the given figure, ind the slopes of the lines d, and da Answers 1-l 2120 3m, 2. Finding the Slope of a Line We have seen how to find the slope of a line when the mclination of the ne is given. We can also find the slope when two points on the line are given, using the following theorem. ‘The slope of a Line passing through the points A(x,, y,) and B(x...) is Let a be the inclination of the lme d which passes through the points AQ, y,) and B(x, 9.) As seen m the figure, mm(ZADO) = m(BAC) = a (caresponding anges) se, AC = xy x, and BC = 1.—ty ‘Now, m the right tangle BAC tang =2C BB and tance Therefore, m EUS 29 Solu [E= 30 Solution En 31 Solu EES 32 =— IA, B, and C are three points on the same line then My = Mo = Me Et ] Find the slope and mehination of the line which passes through the pornts A(0, ~1) and B(2, d. The slope of the ne passing through the points A(0, 1) and B@, 1) 1s, Yy- 0, I-CD _2 em Since my 2 |. then the inclination of the Line as «. = The melmauion of the line passing through the points A(I, 2) an B(-1, a) 16 120°, Find the value of a my =tan120° and m, = tan 60" =a = 23 a= 242N3, ‘Show that the pamts A(4, 1), BO, We have to show that mg = Mge. 21 3-4 My so A, B,C are collinear AG, 1), BCL, 3), and PCO, ky are three pomnts in the coordinate plane. For what value of k does PA PB have its greatest value? tte Anais of Line ad Cicer En 33 Solution Analytic Analyse of Lives ‘Look at the figure. For the greatest value of PA - PB, the points P, B, and A must be collmear Let us thmk why IP, Band A are not collinear, they form a triangle. We know that mn any triangle the difference of the lengths of any two sides 4s Less than the lengths of the thi, by the mangular mequality. Therefore, in APAB, PA — PB < AB. But when P, B, and A are collinear, PA ~ PB = AB. Therefore, PA ~ PB wall have ts greatest value ‘The pomts AC 2, 5), B(-4, 2), and P(O, k) are given. ‘Whats the value of k for the smallest value of PA + PB? Look at the figure. PA + PB has its smallest value when Pas on the same line segment as the symmetry of A ‘with respect to the y-axis, Otherwise, PBA’ will be a iange where PA’ + PB > BA’ (by the tiangular mequaliy) Hence, P, B, and A’ must be collinear: Myy = Mae 2-h _3-k 4-2h=-20 + 4k -6h = -24 haa, Check Yourself 9 1. Find the slope of the line passing through the points A(-I, 7) and B(O, ~3) 2. The inchinavon of the ine which passes through the pots A, -v3) and B¢, 23) 1s 120°, Find 0.9) In the figure, the pomts A, B, and K are on the Line d. Find the value ofa. = En 34 Solution 3. Parallel and Perpendicular Lines a. Condition of parallelism In the coordmate plane, let d, and d, be two parallel lines. As seen in the figure, m(ZCAA,) = m(ZCBB,) (corresponding angles). ‘The slope of the line d, is m, = tan (ZCBB,) and the slope of the line d. 18 m, = tan (ZCAA,), Consequently, the condition of parallelism is ali, mam, The pots A(@ 1, 3), BQa, 1), CC, a), and DG, a + 3) are given. If the lines AB and CD are parallel to each other, find a Since the hnes AB and CD are parallel, their slopes are equal to each other, that 18 ABI\CD & thy = they 1-8 at3-a +3 30+ °.. 2,80 b. Condition of perpendicularity Loot at the figure. The slope of the line d, 1s. ‘m, = tan o, and the slope of the line d, is tan o, = -tanp. In the night tangle ABC, m=tana,=4° ana Be ae ac Let us multiply both equalities side by side, m, = ~tanps 1 mem, = 20. BC me Fe" ae Consequently, the condition of perpendiculanty 15 aid, om tte Anais of Line ad Cicer EXE 35 Solution EEE 36 Solution Without using the Pythagorean Theorem, show that the points A(3, 4), B(-2, -1), and C(4, 1) are the vertices of a ght triangle Let us find the slopes of the lmes on which the sides of AABC he. 1 Nonce that myc M™e= 5+ 3) = so BC AC and therefore ABC is a right triangle, ‘The pomts M@k + 1, ~9), N(, -2), and KCI, -3) are given. If the lines MN and NK are perpendicular to each other, find k ‘Smee MN and NK are perpendicular, the product of their slopes is —1 Ma hg, = =I -245, 349 ¢ Tay aS qige-h wo-840h=2, and so b=! Check Yourself 10 1. IAB IICD, find the value of k, where A(-k, 3), B(-7, 13), C@, 1), and D-# ~ 1, 6) 2, The points B(-3, 10) and C(-9, -2) are the endpoints of the hne segment BC. The ne from the point K(m, m ~ 4) to the midpomt of BC is perpendicular to BC. Find the value of m. 8. Show that the points A(7, 1), BCI, 4), C7, 0), and DQ, -3) are the vertices of the trapezoid ABCD. (Lint: Prove that there exist (wo parallel Lines on which any (vo opposite sides of Uie trapezoid lie) 4. In the figure, d, 1 d.. Find the value of k Answers 13, 10 = Bio 428 1-3 7 4 Analytic Analysis of iney C. EQUATION OF A LINE Every straight line can be represented by a linear equation of the variables x and y. Conversely, every near equation of x and y represents a straight line 1, Equation of a Line in Poi it-Slope Form ‘The equanon of a line which passes through the pomt A(x,, y,) with slope mis, yy, = mee —x) Proof In the coordimate plane, let us draw a line d ‘with slope m, and let A(x,, y,) be a point on the line Suppose that P(x, y) € d 4s a pomt which vanes along the Line In the figure, m(ZCAP, m(ZA,DA) (corresponding angles). Smceoas the mchnation of the line @ then m = tan and m the night tangle CAP we have m=tang= aC yw xa ‘Therefore, . and consequently, the equation of d is y — y, = m(x —x,). STL BV ware te equation of the tne which passes through the pomt A(-1, 2) with slope m = 3. Solution By substtutng m = 3 and ACY, y,) = ACL, 2).4m the equation y ~ y, = m(x ~x,), we get y-2=3@-(1) y-2=3r43 y= 3x + Dis the equation of the Line. ‘We can also waite the equanon as 3 +5 = 0. = 93 rats Aisi nd Cre SENT 3G rina the equanon of the line with mehination 45° which passes through the pomt K(0, 3) Solution Since o. = 49° then the dope m = tan 49° = 1 By substitunmg m = 1 and K(0, -3) m the equation, ym = m@-x) y-C%)=1-@-0) y +3=xandwe gety 3 Find the equation of the line which is parallel to the line joining the points K(-1, ~4), L(S, -10), and which ntercepts the x-axis at 8 10-4) _-6_ 3 -5--D -4 2 Solution Letdbe theme. mg, Since d || KL, m, Also, af the line mcercepts the x-axis at 5, it passes through the point (9, 0). Hence, the equation of the hne d with slope m, = 2 which passes through (3, 0) 1s y-y,=m@-x) ro= 2 em Note ‘We can wnte Uie equation of a Lne mi two different fons: y = mx +n, and ax + by +e where m,n,a, bce RB. 4O rina ine equauon of the perpendicular from the pout K(5, -3) to the Line with endponts AGA, 4) and BQ, 3) Solution The slope of the Le passing through the pomts A(-1, 4) and B(2, 3)15 3-41 =D 3 Let m, be the slope of the perpendicular ne, then m, +m, = -1 (condition of perpendiculantty) =i 3" Therefore, the equation of dis y-(3)=3@-8) y+3=3x—19,s0y =3v-18 or 3x-y-18 Avalyic Alysia ofimy lg = m, =1, som, = 3. SY AV a mangle asc nas vernces ACI, 1), BG, 3), and CC -1), Find the equation of the hine on Solution = = 42 Solu which the altitude of the triangle ABC through the vertex B lies, ‘The slope of AC as my. = since AC L BH, Myo May Dey May 1 and we get ‘Therefore, the equation of d3s_y- y, = m(@&- x) 1-3) and x Check Yourself 11 1, Find the equation of the fine with incdmation 190° which passes through the point AC-. 5). 2, Wnte the equation of the Ime passmg through the point M(-9, 1) wich 1s parallel to the line with endpoints K(7, -1) and L(0, 3). 3. Find the equation of the hne whose y-intercept 1s -5 and whieh 1s perpendicular to the Ime wath endpoints P(-I, 6) and R(-2, -3) Answers 1.2N8x + 6y-34+VB=0 24xtTyt13=0 3.xt Oy t+45=0 In the line equation y = mx +n, mas the slope and nm isthe y-antercept of the hne ‘The distance of the y-intercept of the ne y = (a — I)x ~a” from the ongn 1s 9 units. Find @ and draw the graph of the hne (a € R°) m= a1 Glopey n= ~a° (y-intercept) If we take ~a = 9, then a” = -9 which is not possible, so the y-mtercept must be -9 and if -a=-9 @=9anda=3 @eR) Som=a-1 2 3-1 Hence, the equation of the bne 3s y = 2x ~ 9. pratt Anaya of Lines and Cite ES 43 Soluti x 4 Solution Analytic Analyse of Lives Note ‘To find the y-intercept of a line, substitute 0 for x in the equation of the line, ‘To find the x-mtercept of a line, substitute 0 for y in the equation of the bine. Draw the graphs of the lines y = -x + 5 and y = -x on the same coordinate plane, Notice that the slopes of both lines are m = -1, so they are parallel to each other. Ifthe graph ofa lmem the form = mx 1s shifted m unas up (when n > 0) or n umits down (when n < 0) along the y-axis ithout changing the inchnation, we get the graph oty ‘We know that y = ~ 38 the second diagonal. First. draw the graph of y = x and shift the le y = —x Wve umits up along the y-axis to get the graph of y = -¥ +5, mx + 0. In the figure, ABCD is a square. The hnes d, and d, unrough the origin pass through the pomts D and C, 3 respectively. If the equation of d, is y= 2x and the a equation of d, 1s y = ax then find the value of a. 3 3 3x is m=. The slope of the line y= > 5 In the right triangle DOA, tan(zpoay = 42-3 20 z hk and AO = 2k and each of the sides of ABCD 50 AD = 353k From the night tangle COB the slope of d, can be found 3k tan(ZCOB) 5, 5 3 ‘Therefore, the equation of dis y=—x and a= QS, Find al the possible values of k such that the points AC, 2) and BQ, 4) ate on diferent sides of the Line with equation 2x + 3y +k = 0. ‘When we write 2v + 3y + k= 01m the form 2k y= mx +n, we get y=— 3° 8 Consider that the slope of the Line an this form is m 3 Let us draw the lines d, and d, with the slope 2 passing through the pots A(-I, 2) 3 and B (3, 4), respectively To find the exact equations of d, and d, we must find the appropriate values of bt for each line. ‘The point A satisfies the equation of 2 ky 4 33 By substituting AC-1, 2) im the equation for |, ary (ew), we get, 4 Sod ss y= Fe S and n, = 5 Smnilarly, the point B sausfies the equation of dy wa nZe By substituting B@, 4) in the equation for (x, 9), we get, by2-t8 50d, sy=2e+6 ant 28 Avalytic Analysis of Lines and Citler ‘Now, the points A and B will be on different sides of the Lines d, and d, when the y-mntercept 2M seteinean and, (tr the Bp of the Line with equation = —= kh That is, m<— 5 < me 4G wine neue, ne equation of tne une dis y = 33x + 3¥3 + 3. What is the area ofthe square OABC? Smee AB = BC, the point B has the coordinates (—a, a), This pomt sausfies the equavion of d because B(—a, a) 1s on the Lme d By substituting BC a, a) m the equation for d, we get, y= Bx + 843 a= Ba) +343 a+ Ba =303+1) ad + 13) = 303 + 1) a=3 Hence, A(@OABC) = 3° = 9 square units. Avaitic Armpit ofime lg = Check Yourself 12 1. Draw the graphs of the lines y = -2x —1, y = -2x and y = -2 + 1 in the same coordinate plane and compare them, 2 Find the slope of each line. ax-3y tl bys Brt 1 oS + dy =—w 3.1m the figure find the area of the tangle ABO formed by the lines y = 2, y= my and the y-aus, m terms of m. Answers 3. 2/m 2. Equation of a Line in Two-Point Form The equation of the lane passing through two fixed points A(x, y,) and Bory #18 wan % Proof Let a be the mclinaton of the kane d which passes Uhrough the points AG, 1,) and BCs 43. Then the slope of the kme ds on | m=tane. = moe mH We know thatthe equation of hetine with agven pont and slope is | yy, = mO-X). wer So y-4, “(e=¥). Yt xo Hence, the equation of the ne passing through two pomts is 4 rand ine equaton of nene wich pases tnouth the points ACI, = ge rts ified Cre 3) and BOS, 1). Let us subtitute the points A(-1, -3) and B(S, 1) am the formula. xx, xal “1 _xdl 6 By +9=2042 2x-3y-7=0. SET 4G the pomes a. -8), BCA, 1), and Cm, 2) are on the same lune. Find the value of m. Solution 1 The equation of the lime AB ts y+5_ x-2 eS 2 ~8y-15= 6x-12 6x43y43=0 2Qx-tyt1=0ory Since C(m, 2) lies on AB, it satisfies the equation y = -2x so 2=-2m-1 3=-2m 3 m 3 Solution 2 Notice that we can solve the problem by using the slope formula. Smce A, B, and C are collinear, ys = Mae 2-1 mal 6 a Amt Ames of Linge gg = EE 49 Proof IfACI, 2), BQ, 1), and C(O, 4) are the vertices of ABC, find the equation of the median durough the vertex A. Let N be the midpomt of BC, then mts coordmates are 240 144 =na) 4 ra he 1 Mh) NC AN is the median and it passes through the points AGI, 2) and NG, 5), costs ejuation a ¥ way an 5 2 a a y t and so the equation of the ine as x — 4y + 9 = 0, Check Yourself 13 1. Whats the equation of the me oming the pots ACI, 7) and B-2, 6)? 2. a. ind the equation ofthe me passing through the poms 4, 0) and B(0, 3) b. Find the value of kif the above line also passes through C(k, 3). Answers Ly=x48 2a3x¢4y-12=0 D2 oe Since A(@, 0) and B(0, b) are the mtercepts of the line , from the equation of a Line whose two points are known we have, ‘This is called the mtercept form of the line equavon Avalytic Analysis of Lines and Citler Solution Em 51 Solution Solution Analytic Analyse of Lives Find the equation of the bne passing through the points A(-5, 0) and B(0, 3) ‘The xantercept of the ne as a = -9 and the y-ntercept of the hne is b = 3, so Hence, the equation 1s 3x —5y + 19 = 0. Find the equation of the line containing the point P(-3, 2) which passes through the pomts (0, @) and (a, 0), equidistant from the origin. Lane d intersects the axes in the second quadrant. ‘As seen m the figure, point P(-3, 2) satisfies the equation of d Find the equation of the line which passes through the pomt PC, 2) and whose segment between the axes is divided by the point P in the ratio >, as shown in the figure. : ‘We need to find the x- and y-antercepts, 1.¢. the value of @ and b Look at the figure. The yalue of a: The abscissa decreases five units over a length 3h, as shown in the figure, Therefore, over a length of dk the abscissa will 9-9 ums, s0 a =—2%. 33 3 The yalue of b: ‘The ordmate mcreases two umits over a length of | 2k. Therefore, over a length of dk, the ordinate will increase five umts. 50 3x, Sy 35" 35 decrease 9 B16, ») Hence, + Sx -9y-+ 29 =0 as the equation of the line. 25 3 ‘= FEZ 5B The area ofa triangle formed by a tne and the coosdinate axes sx square unit and the Solution Jength of the segment intercepted by the axes is five unis. Find the equation(s) of the Line. ‘AB = 3 umts ‘A(AAOB) = 6 squaze units As shown in the figure, « and D are te intercepts of the line on the axes. acaaos) =“ =6, ab = 12 ana AB =a? +b; a +b = 20 (a+ by —2ab = 29 25 $2.12 =49 -Torath=7, @+ny 7 > a-Tat12=0 (a-3y(a-4)=0 1) a, =3andb,= 2 = 4,0 242-1 or ay + 3y-12=0 (ay. 3 a4 2 xyye 2) a, = 4 and b, = 7 = 3,50 F42=1 or Sx + 4y-12=0 Cage I: 2 “7 > ab 47a+12=0 (a+3)(a+4)=0 2 Da, 3 and by = ~4, 50 4y + 3y +12=0 (a). 2 a Bx 4 4y 412=0 Gd). So the equations of the lines are d,, dy d, and d, 2) ay = ~4 and by Avalytic Analysis of Lines and Citler 54. Find the area of ne regon formed by the lines x-y + 2=0, 2x + y-2= 0,and Solution Analytic Analyse of Lives yt2=0. To spectfy the region, let us first draw the graphs of the lines by finding the x- and y-mntercepts of the Lines. dsx-y+2=0 xy 0/2 @2 210 (20) dg Qe + y-2= [a 2 0,2 o «ao dg y +2=O0ry B(m, -2) sausfies the equation of d,: m — (2) + 2 = O and m = 4. C(n, -2) satisfies the equation of d.: 2n + (-2)~2 = O and n = 2. Therefore, we can see that AD BC+AD_6-4 units and BO=442 units. Therefore, A(AABC) 12 square ums, Check Yourself 14 1. Find the equation of the lane whose x and y-antercepts are ~4 and ~3, respectively. 2. Alme passing through the pout P(2, 3) mtersects the x- and y-axes at the points A and B, respectively: If PA = 2. PB then find the possible equations of the line AB. 3. The x- and y-mtercepts of the ne aze 2. and -3, respectively. Venfy that the Lne passes through the point P(4, 3). 4. If the intercepts of a line Sx + 12y ~ 60 = 0 are A and B, find AB. 5. Allme has its x-and y-ntercept at a and b respecuively, such that a + b= 9. If the Lne passes through K(4, -3), find the possible equations for the ne. Answers Lax t4y+12=0 23x +4y-18=00r3x-4y+6=0 413 5. 3x + 2y-6 =0orx-2y-10=0 4. Equations of the Coordinate Axes We have seen how to write the equation of a line. What are the equations of the two coordinate axes? 0, We can = ‘The inclination of the x-was is o: = 0°, so the slope of the x-axis is m = tan 0° also see that O(0, 0) 1s on the a-axis, Hence, the equation of the x-axis 1s ya, = mx —x) y-0=0-(v-0) yea (On the other hand, the inclination of the y-axis 15 B = 90°, so the slope is not defmed, and therefore, m = tan90*1s undefined, Since the expression 21s undefined, the denominator must be 0. Therefore, the equation of the y-axis 1s Recall that the ordinate of each point on the x-axis is 0, that 1s, for every pomt (x, y) on the wands, y = 0. Also, the abscissa of each point on the y-axis is 0, that is, for every point (x. y) on the o y-axis, x 5. Equation of a Line Parallel to a Coordinate A Let us draw a hne through the pot B(0, b) parallel to 7 the x-axis, The melination of the line 1s o = 0° and 1s 4c0,0) Pst) slope 1s m = tan 0" = 0, mex) 0 @~0),s0 the equation of the hme dis ya Let us draw another line through the pomnt A(a, 0) parallel to the y-axis. ‘The inclination of the line, B = 90° and its slope is undefined, y-y,= ma-x) y-0=m@ ~a) m= tan 90° Considering m 3s undefined, the denommator of the expression —Y— must be 0, that as, x -a Hence, the equation of the line 1 15 eccal tte Anais of Line ad Cicer Solution EES 56 Solution Ea 57 Ean 58 Solution En 59 Analytic Anaya of Lis ‘Show the following lines in the coordinate plane. b. 44 o ay=3 D. x +4=0,50x= Show the lines passing through the pamt A(1, 3) which are parallel to the coordinate axes. ‘We need to draw a figure Like the one opposite. We can see that the nes arex = 1 andy = 3, Find the equation ef the horizontal line passing through P(5, -2). ‘Smce the hne 1s horizontal, 11s parallel to the X-axis. It cuts the y-axis at B(0, -2) and so the equation of the line is y = -2 ory + 2 = 0. Find the equation of the vertical ne passing through K(-7, 3). Smice the ne is verbal, 18 parallel to the y-axis. It cuts the x-axis at BC equation of the line is x = -7 or x + 7 = 0, 0), and so the Find the equations of the diagonals of the rectangle whose sides are on the lines x x= Qy=-2andy=4 nv Let us draw the graphs of the four Imes in the coordinate plane. As we can see mn the figure, the vertices of the rectangle ABCD are AQ, 4), BEL, 4), CCI, -2), and DQ, -2). ‘The line d, passes through the pots BC, 4) and D(2,~2), so the equation of d, 1s xt 21 3y-12 =-6r-6 Gx + 3y-6 = 0, sowe get 2x ty-2=0. The Line d, passes through the pomts A(2, 4) and C(-1, ~2), so the equation of 1s, 2 you, YY, and -3y + 12 =~6r +12 ~ 8y = -6x, so we get y = 2x Check Yourself 15 |. Find the equations of the horzontal and vertical Lines which pass through the point Pe-9,-2) 2. Show the region bounded by the lines x -3 = 0.x +2 =0,y +4 =0,and y~ the coordinate plane. Find 1ts area, om Answers 2, 28 square unats D. FINDING THE SLOPE OF A LINE WITH A GIVEN EQUATION Recall that we ean write the equation of a Line in two different ways: 0 y= mx + mand ax + by + For example, y = 2x — 7 and 2x — same bne, —7 = O are two different forms of the equation of the The slope of a line given m the form y = mx + m 1s m (the coefficient of x) But to find the slope of a line given in the form ax + by + ¢ = 0, ts useful to convert it to the form y = mx + 0 ax tby te =0 by =-ax-0 Analytic Analyee of Lines and Cites EE 60 Solution EE 61 Solutio Analytic Analyse of Lives with y = mx +n: Consequently, the slope of a line gven in the formax + by +¢ = 01s m Conclusion 1, The slope of a lme given im the form y = mx + n 18 m. 2, The slope of a line gven.n the form ax + by +¢ = Os ‘The equation of a line with the inchnation 135° as (2k — Ix ~ (k + 2yy +4 = 0. Find k. and —k-2=2h-1 ReaD x Given that the lines 3y are parallel, find the value of a. =p andax + The slopes of parallel lines are equal, so let us find the slopes of the given lines. z, 2 y=2x—2pand m, a; Boxy dy: ax+5=3y ax~3y+5=0and m, since m, 2=" andso a6, 3 GQ Thetines omx—3y = 1 and y = 1—2e are perpendicular. Find m. Solution The product of the slopes of perpendicular lines ts equal to —1 Let us find the dopes ofthe nes. a 2mx ~ay = 1 2mx ~3y-1=0 im, =—2m = 2m a) 3 om A, s0 o 3 2) EXO GZ the tines a,: 3x + y = 4, dys x ay +7 = 0, and dy: bx + 2 + 5 = 0 form three consecutive sides ofa rectangle. Find a and b Solution As shown m the figure, d, dy. d,1d,,and d, 4 4, z 4,|/ 4, 50m, = my and so -3=-4 sob=6 4,1 d, 501m, -m, =-1, 50 -3-4=-Iand so a=3 ' ° a Stet SEE G4 find the equation of the locus of all pomts of the form AC ~ 81-2) Solu Let A(x, y) be a point on this line. ac, p= ACF, 2-2) a From (1) and (2), 8 6x-3=5y+10 6x-y-13=0. ‘Therefore, the pomt act =3t-2) represents a locus with the equation Gx ~ Sy ~ 13 = EXIT GF Fin the equation ofthe ine perpendicular tothe line 2x + 3y +7 = O which intercepts the pecs at-3, = a rats Aisi nd Cre Solution We know that the product of the slopes of perpendicular lines 1s equal to ~1 ‘The hne passes through the pamt (0, -3). So the equation 1s yoy m(x ~ x) Y= CH= 5-0 andy +3 w-3 vie 80 y 6G Find tne equacion of the ne paatel to the ime 2x + 3y = 5 which has the same y-antercept as xtyt4=0 The slope of the Ime 2x + 3y 1s m 2, 3 The yantercept of the bne x + y +4 is 0, -4). 2 The slopes of parallel lines are equal, so the slope of the desired line 18 also ~ The y-mtercept 1s -4 and the equation is y = mx +m ye Beet rae 48) 4 Check Yourself 16 1, Thebnes x + 4y-1=Oandax + by +e sk ae perpendicular to each other, Fund “> 2, Find the equation of the line parallel to the lme 2x + 3y = 9 which passes through the point P(-1, 3), 5. White down the equation of the lime perpendicular to 3x + 8y = 12 which passes through the pomt P(-1, -2). 4, The slope of the line (2k ~ Ix ~Sky ~ ois 5 rnd te. Answers 1-3 220+ 3y-7= E. RELATIVE POSITION OF TWO LINES The lines d,: a,x + by +¢, = Oand dy ax + bay +, 3 8x-3yt2=0 44 are given, i. then the lines are comeident, that 1s, d, S then m, = ms, that is, dll dy then m, # m,, that is, the nes intersect at one pomt, Analyte Anaya of Lingy gg = [EE 67 Solu ‘The point of intersection of two lines is equal to the solution set of the system of linear equa- ons ax thy te,=0 axt byte =0) for the equations y = m,x + mand y = max + ny Determine the relative position of the Imes x ~ 3y +2 = O-and 2x ~6y +4 4a 3a? (tne ratios of the cot a) Soap Hg (te satos ofthe coefficients are equal) ‘Therefore, the lines are coincident (on the same line) ‘The lines x + 2y—2 =O and (k—1)x + y +4 = Oare parallel, Find k and draw the graphs of the Les, 4, ds, 50 n=3. 2 Letus draw the graphs of the Lnes: ape t2y-2=0 ds te eysa=0 xy xu ofa 0|-4 alo -8| 0 tte Anais of Line ad Cicer EEZUIIEN 69 rans the iverseenon point of the mess + y + 1=Oand 2e-y +2=0 Solution When the equations are solved simultaneously, xtyt1=0 Qx-y+2=0 3x 43=0 x=-l > -ltyti arn The mrersection pomt of the lines 18 therefore 1,0). ‘We can also find the intersection pomt by sketching the graphs of the Les: dgxtyt1=0 Q-yt2=0 xiy x |y of-1 ofa -1| 0 -1]o 7O inure neue, na 2. the coordinates of the point A. b, the area of the tangle AOB. 0) Solution Furst, let us find the equation of a: Eybe o aint nya 2 a, Pomt A(a, b) is the mtersection pomt of d, and d;, So by the simultaneous solution of the equations, wy 0 > vt2-Cay-4=0 -x-4=0 4 and y So (-4, 4) are the coordinates of the point A. b. For the area of AOB, since OB = 4 units and the alutude from Uus side is also 4 units, then Acanony = 14 — gua ui Auli Amy of ime lt = Es 71 Solutio EE 72 Solution ae = = Do the lines x —2y + 6 = Oand-2x + 4y +6 antersect? -y +6 2x + 4y +6 =0 Qe ~ dy +12=0 2x + 4y +6 =0 1 =0 > 8=8 4d, 0d, = @ so the lunes are parallel and therefore do not intersect. Notice that the ratios of the cofficients are and d, || dy which means that m, = m, Find the intersection point of the hnes 3x ~ 4y + 12 = 0 and ~Gx + 8y~24= 0, Bx - ay +1 Gx + By - 24 6x — 8y +24 =r + 8y - 24 o=0 + S=R There are unfimitely many solutions to Ue system. So the lines are comeident. 3-4 2 1 Notice that 2 —=4 hence, d = ds ae hence, d, = d, Check Yourself 17 Determine the relative position of the following pairs of lines and find their mtersection points, if they exist 1 -Qetytd 2y 2 Te-3=—ay 8y—10 = 14x 3. x-8y-13=0 * +3y-1=0 3 ty Answers 3, They are intersecting lines and the intersection point 1s 3,4) Analytic Analyee of Lines and Cites TB the equations 3x — hy + 6 = Oand (aye + 29-4 = 0, Find kat 2 the Lines are perpendicular. b, the lines are parallel. ¢. the lines are eoincident (h=5) 7G, The lines ax + 2y— 4 = 0 and 2x + by + 6 = 0 are perpendicular and intersect exch other on the waxs, Find (a,b) Let P(O, h) be the mtersection pomt of the lines. PQ, k) satisfies both equations, so 4-0 +2-h-4=Dgvesk=2and | 2045 46 =O.gvesk = 2 {7 yp me We know that a = -b, soa and (a,b) 3, -3) SESE FH m the figure, the equanon of a, is y = 2x ~ 3 and the equation d, is y = x + 0. Given AdOCE) = 7 square units, nd the value of n Analytic Anaya of Lis =— Let us find the pomts A, B, C, D from the gven equations C and D are the intercepts of the lmne y = x + m, thats, forv=0, y CO. m), fory = 0, x=—m Den, 0) 3 3 A1s the x-ntereept of the hme y = 2x ~ 3, that is, for y = 0, X= 5-andso AC, 0) B is the intersection point of the Lines. By solving these equations simultaneously, yextn = rtn=2x- y=2x-3 ' 8 wemes so ueste | pa t3.ant3) y=2ant3, However, the given area AOCB is the difference of the areas of AABD and ADOC, i.e. A(AOCB) = A(AABD) ~ (ADOC). AD=n4 3 2043 2 i icin = A(AOCB) = 5+ AD “BH - 50D -0C op=0c=n 23 _ 1) 2n43. Bas +Qn+3)-n- a To gh mk mal 23 _ 4n°+12u+9-2n 2 2 Qn? + 120-1 n+ 6n— (+71) =0 Torn Since m 1s postive, n = 1 Check Yourself 18 1. The hnes (6 - iy + 2x -3 = 0 and mx —~ other on the x-axis. Find kt —m. 2 are perpendicular and mtersect each. 2. Wnite the equation of the line which passes through the origin and the intersection point of the hnes 2x -y ~4 = Oand 3x +y-3 = 0. Avalytic Analysis of Lines and Citler 3. Inthe figure, dy: x + ~8 = 0, dary —3x = 0, and y a x-y = Oare gven a ‘What i the area ofthe triangle KOL? e Answers x 19 20s 3.8 square units F, FIRST DEGREE INEQUALITIES IN TWO UNKNOWNS In the coordmate plane, a lme separates the plane into two egons. Smce the equation of the ine asax + by += 0, then we can desenbe the two regions using the inequalities ax + by te <0 andax + by +¢>0. To show the ahiton se ofan ineqaliyn the plane, este dw the wap of the xb After that, we take an arbitrary point from one of the regions. If the point satisfies the gven Dneqaby; he mens hat the epon where we ook the pnts he suton st, others he ore gn il be the sition set ofthe pen megs, emonber thas he equay cons the sgn <> ton theae wel not ne the salution st, Dut the mega conta he signer = then the ne lis neh m ho) tnesetuton set oF ne equa We show nicked nes stright ine, and exchided nes as a dashed line, "7G show the solution set of the anequabty x + y -2 > Osn the coordinate plane. Solution Since the inequality contains the sign >, we show the line x + y -2 = 0 asa dashed line To decide the appropnate region for the inequality, we substitute O(0, 0) 4am the Inequality. 0+0-2>0 + 2>0 This 1s incorrect, so the other region is the solution set, This is the shaded ‘region m the figure. Avalyic Aral of my tg = 77 wore ne mequatty of the graph gven i the Age Batonsan4o-0, Substituting O(0, 0) m the equation gives 0-2.042=250. So the spe must cont thes A the ne ane, 0 he nea ‘Therefore, the inequality is x —2y + 2>0. Solution The equation of dis 7G draw tue graph of the system of mequaliues given below, x4 2y20) -y-4<0 Fist we draw the nes x + 2y = 0 and 2x—y—4 = Os astraight line anda dashed Lune, respectively. dx +2y =0 a: ey toe ‘To find the appropriate regions, we substitute 1, 0) m the first mequality, that is, 142. 0=1> 0, which is true ‘We substitute (0, 0) in the second inequality, which gives 2-0-0 -4=~4<0, which is tue for the given conditions. ‘We shade the two regions, and their intersection as the solution set of the given system. 7Q show that the points A(, 8) and BC-3, -2) Me on the same side ofthe line 3x - 7y -8 = 0 Solution Let us substitute the pomts A(3, 5) and B(-3, -2) im the line equation: 3-9-7-5-8 =-34and3-(3)-7-(2)-8=-3, Both points A and B satisfy the inequality 3x - 7y - 8 < 0. Hence, AG, 5) and BC3, -2) le on the same side of the hn. = 86 rats Aisin nd Cre [EEE BO stow the satution set of the system aes 1m the coordinate plane lyl22, Solution a. |x[>1 ifx20, x>1 () ifx<0,4>1 = x<-1@) b Iyl22 aty20, y22@) ify<0, -y22 3 ys2@) ‘The combined solution ts $ = $, US, US, US, shown in the figure Check Yourself 19 ' 1. Write the inequality of the graph given m the figure. 2. Show the pomts m the coordmate plane which satisfy the system of mequabities x+y s3| axty>3i Answers 1. Bv + y-6>0 G. BUNCH OF LINES (OPTIONAL) bunch of bnes: In the coordinate plane, a set of the lines passing through one common pomt is called a buneh of lines. Avalyic Aral of img lg = EU 81 Soluti En 82 Solution Let dy: ae + by +c, = O and dy ax + by + c= O be wo ‘nes belonging to a buneh and A be the mtersection point of all the Ines in the bunch, The equauon of the bunch of Imes 1s, wnitten asa,x + by +e, + k(ax + by + ¢,) = 0 (Ite R). More generally atk @=0 ‘The point A in the figure is the simultaneous solution of the equations of d and d., Find the common point of the bunch of lines given by (Ik +1)x + Qk — ly + 3k —4=0. Let us ty to waite the given equation in the form d, + ke- d, = 0. e+ Dx + Qk-y + 3k-4=0 kx tx + Qky-y + 3k-4=0 wy 4 +k + 2y +3) = 0, sod,x—-y 4 = Oandde x + 2y +3=0, ‘The mtersection pomt of the lines 3s now the solution set of d, and x-y-4=0 x42y+3=0, ° ; a0 3987 & soareuneneeuon ann a Find the equation of the line which passes through the common point of the bunch of lines given by the equation kx + x + ky —2y + 3k—2 = 0, and which is perpendicular to the line Quay +1 =0. First, let us find the intersection point of the Lines. dex +x + ky —2y + 3k-2=0 x-W-2th@ty +3) -2y-2=0 xty43=0, 4 4 x= —$s0.a(-$,- -3y= 5,4 3 3 since the hne a perpendicular tthe ne 22 —3y +1 =O and m 3 rapa sao aia a - - There tie equi of th tine psig trash A(-4 9) wt logs m= 3 yy, = m@-x,) yt2=3erd me ge or 9x4 6y+22=0. Avalytic Analysis of Lines and Citler We can also find the common point of a bunch of lines given by ax + by +6, + k(ax + by +e) = Oma different way The simultaneous solution of the equations ofat least two lines obtained by substituting arbitrary values of k will be the mtersection point of the bunch GB _ rund the common pomt ofthe Lines given by the equation kx -28 + 2hy + y + k-2 Solution Fork =0,-2x + y—2=0, and for k = 1, -x + 3y—1 = 0. These are two lmnes in the given bunch, and their simultaneous solution 3s oe + y-2 +3y oy +5 =0 y=-Iand y= Therefore, the mterseetion point 1s ACL, 0) Check Yourself 20 |. Find the mterseetion pomt of the bunch of lines given by the equation Gk -2x + dk + y—k-3 =0. Find the slope of the line belonging to the two bunches (2k -3yx + (hk -4)y-3k-G=0 and (ke + 5)x + (Tk—S)y + 3k + 10=0. Answers 317 163, 1% Fs Analytic Analysis of Lines EXERCISES 1.2 A. Trigonometric Ratios of Angles . Find the tangent values of the acute angles. a a= 30° Dd a=40 6 = 60" 12, Find the tangent values of the obtuse angles. aa=120° ba=13° © 3, Find the tangent values of the angles a0” b, 90° ©. 180" B. Slope of a Line 4. Find the slope of the Ime with the given inclination, a 0 =30 D. a= 120" © = 90 aaso 5, Find the inclination of the line with the given slope. a m= b ma Bu-1 m=o 6. Find the slope of the hne passing through the gaven poms. a. AQ,-1) b. K@,-2) BC3,2) LG, -1) ©. PC, 0) a. MQ, 7) RQ, 2) NCI, -3) — 7. Find the inclination of the line passing through the points A(-3, 0) and BO, ~/8) 8. The points ACI, 1), B@, 3) and C¢k, 5) are collinear. Find k 9, If the metmation of the Ime passing through the pomts ACJ, 1) and B(k, -2) 1s 150°, then find k 10, The pomts AG. -5), B@, 1) and Com, -1) are given. IFAB 1 BC, then find m AL, The points A, -6), BG, 2), C(a, 9) and D(2, a+ 1) are given. IfAB II CD, find a C. Equation of a Line 12, Find the equation of the line passing through the ‘Given point with the given slope a. A@, 0) b C@,4) — & KG, 0) m=-1 m=V3 m=0 18, Find the equation of the line passing through the ‘gven point with the given inclination a. AC2, 3) >’ BR.) a= 150" = 49" ce. C(-3,-4) b. D1) a= 120° a= 90° ‘14, Find the equation of the line passing through the point A(4, -6) which is parallel to the line jommg the pons 8-2.) and Analytic Analyee of Lines and Cites 15, Find the equation of the Ime passing through the onigin with an inclination 120°, 16, Write the equations of the Imes passing through the omit A(@, -2) which are parallel to the x- and y ais, respectively, 17. Wnte the equauon of the perpendicular bisector ofthe line segment AB with endpomts A(4, 7) and, BQ, -1). 18, Fund the equauion of the line passing Uarough the point P(-1, 2) which is parallel to the Lime jomng AQ. 4) and BQ, 3) 1G. The pomts A(-2, 4), BQ, 1), and C1, -3) are the vertices of a mangle ABC. Find the equation of the altude of the mangle through the vertex A. 20. Find the equauon of the Line passing through the given two points, a. AC2.1) b. C(-3, 0) BQO, 3) D(A, -2) ©. KO, -2) a Ma, 7 L6.-) NCL, -3) Bll. AC 10, 12), BEG, 4), and CC 4, ~ 8) are the ver tices of AABC. Find the equations of the Lines: passing through the medians of ABC 22, For t © R, find the equation of the locus of all points with coordinates of the form A(Bt —2, 1+ 1) 23, The pomts A(-2m, 0) and BQ, mare gven. Find © the equation of the locus of the midpoint of the Line segment AB. Anny Anaya 9f Lingy 9A, Fmd the equation of the locus of all pomts which. are equidistant to the pomts AC9, 2) and BC, 4) 25. Find the equation of the le which passes through the pomt A(-2, 3) and intercepts the x- axis at 4 26. Find the equation of the line which passes through the points A(2, -5) and intercepts the yeas at -3. 27. If the Ime Z=1 passes through the poms a A(2, -19) and B(8, -9), find the values of a and b 28. Write the equation of the line shown in each graph. 20, Fund he «- and y-mmercepts of the following Lnes and draw their graphs. a. Qe 3y-4=0 bo x + 8y-2= ©. -Bx-yt1=0 d 3x-6=0 e. 8y +25=0 ‘= — 30, Find the equation of the kne whose x and yeantercepts are -3 and 4, respectively 31. Find the equation of the line which passes Unrough the pont P(3, ~4) and makes a. equal intercepts on the axes, b, mtercepts equal in length but opposite in sign on the axes. 92, Find the equation of the line passing through the point PG, -2) which has posiove mtercepts on the x- and y-axis.in the ratio 4 : 3. 38, Find the equation of the line passing through the point A(4, -2) af the sum of ats x and y-mtercept is 3 units, 34, Find the equation of the line which passes through the point ACL, -3) fits x-intercept s twice as long ass x-intercept 35, Write the equaon of the line which passes through A(@, 3) if the product of ats x- and y-intercept is 2 36. Find the equation of the hne perpendicular to the 3 Line y= 2x with x-intercept at 19. 37. The area of the mangle formed by the Ime ON + 12y + k=O and the coordinate axes 25 30 square units, Find the positive value of 38, Find the area of mangular region bounded by the Imes x-y + 2=0,2x + y-2=0, andy +2=0, 99, Find the area of tangular region bounded by the lines 3x + 2y - 6 = 0, x + 2y - 6 = 0, and the r-axts, D. Finding the Slope of a Line with a Given Equation 40. Fund the slope of each ne. a de-3y + 1=0 bow +3yt1=0 ce. Bx + 6y-7 @ 4-2x=0 ©. 5y-2=0 AL. The Lines 2m — Dx—my + 5 y= mx ~3 are parallel. Find m 42, The lines 5x ~6y +7 = 0 and mx + (m + Dy —5 are perpendicular. Find m. 43. In the figure, find the value of a. 44, Tund the equation of the line passing through the point P(2, 4) which has the same slope as the Line way +3=0. 45. A le passing through the points AL, 2) and. BCS, 4) 1s parallel to the Line kx + 2y + 6 Find k. E. Relative Position of Two Lines AG. Ifthe lines kx + 8y~6 = 0 and 2x + 4y + are coineident, find the value of k +. tte Anais of Line ad Cicer 47. The kes (m + 2x + (m= Iyy = 4 = 0 and 2x + y—3 =O are parallel, Find the value of m, 48. Determine the relative positions of the following hee lines. Av ty +3=0 dg Wx -3y-9 = 0 dy 8x-2y 41=0 49, Find the intersection pomt of each pair of hnes, and draw their graphs, 9 a Seay + xty-5=0 +3y z X4ay 4320 3 ee + 2y = 5 Te Ly = 39 £60, For what value of k do the hnes © @k-1)x + ky-6 = 0 and kx + de Dy 4 the y-ans? 0 mtersect each other on 51. Find the area of ” © the mange ABC. Analytic Analyse of Lives F. First Degree Inequalities in Two Unknowns 52. Show the solution set of each mequality as a graph. a Qe +y-220 bx-3y+1<0 ce. By-y 4 4<0 4. Sx - 4y-20>0 8. Show the solution set of each system mm the coordinate plane, ax +2y-250 b. Ixl 2] co year td 4 yo ‘54, Wnte the appropriate inequality for the shaded region of each graph. 3} 55, Wnte the appropriate system of mequalities for the shaded zegion of each graph. 6, Show thar the points A(-1, 3) and BG the same side of the line 3x — y +7 1) hie on 87, Check if the pomts M(2, 3) and NCI, 5) are on the same side of the line y = 2x + 5. 8. Which of the pomts A(1, 1), BCL, 2) and C¢-3, 0) lhe on the side of the Ime 4x + 3y = 5 which includes the ongin? 59. Find the area of the region bounded by the inequalities y 0 4 @ is acute angle tan@ <0 <> 01s obtuse angle, tan® = 0 4 m(@) = 0, sod, |Ids, tan@ is undefined <> m(@) = 90°, d, 1 dy In general, we ean say that the tangent of the acute angle between two hnes 1s, anol | im Tem, +m, When two lines intersect there are two intersection angles @ and fi such that @ + B = 180" tan 0 = tan (180 -B) = tan. m,—m, Tem, m, -tanB Lie © Tem,-m, Father Apphiations or = tan B En 84 [En 85 Solution [Em 86 Solu Find the measures of the angles between the Innes dey = 2x + 4and dy y = 3x1, m,=-2 tang = Me m= Tem, tan@ > 0, s0@1san acute angle and the tangent of the obruse angle between these two hnes 1s tan (180° -@) = -tan@ Now, Jet us find the measures of the angles: tan @ = 1, 508 = 45°, © + P= 180", 45 +B = 180", B = 135° Find the measures of the angles between the lnes y =¥/3x ~3 and y = ~V/3x + 3. yo x3 mad | mm -) y=-Bx+3 =m, =-N5) mime TSG) 13 tan = -~V3, s0 0 = 120% and the measure of the acute angle is 180° - 120° = 60°. Be Find the measure of the acute angle between the hnes 3x + y -7 = Oandx + 2y +9 =0. 3x + y—7=0,s0m, = -3, x+y +9 =0, 90 m, 3 mem | |t3|_ [3 tano= [Me | = IememP fs) 8 al o3 tam = 1, so the measure of the acute angle is 45°. Analytic Analyee of Lines and Cites Check Yourself 21 1, Find the tangent of the acute angle between the lines Qe ty +1 =0andx-3y-1=0 2, The les dy: y = ax and dy; y = 3x are given, If the measure of the acute angle between d, and d,is 45°, find a. 5. In the figure, the coordinates of the points are A(, 2), BCA, 0), and C(4, 0). Find the tangent of the angle between d, and d, Answers 17 2aeen4y 33 2 q B. DISTANCE FROM A POINT TO A LINE Further Appcatons “distance ftom. a.poiitt:9 Byme Let A@y 0) be a pomt and d: ax + by + ¢ = 0 be alae, then the distance from A to the hne dis 7 le ea Fel Let the distance of A(x, y,) to the line a ax + by +c = Obel= AH. Take C(s, y=) = ADA. x, = x,andy,= CD isa pomt om the line ax + by +c = 0, 90 ax, +b CD+c=0 b-CD=-a-x,-¢ Es 87 Solution Now, ects the mclnation of and, & = m(ZCBD) = m(ZCAH) (anges with perpendicular sides). AH Jn the night tamale ACH, cos 0= = and AH = AC coscr (1) Now. let's find the equivalent expressions for AC and cose, 4-2), 60 AC= ta ©. $58, s0.ac= 1 +8 +2.) We know seca: = 1 + tan’. 1 Virana, 1 1 = ee Substituting (2) and (3) m (1), AC = AD-CD= 4)~ 50 cosa.= + and tana=m=-5 b hae els 1=AH = AC. cosa, and since Lis the distance, aye _l? TENT S| _ lax, thy, +c = £445 tSlatiatp Find the distance from the pomt (0, 0) to the line x —y + 4 = 0(0, 0) = Oc, y,). Using the formula, Find the distance from A(S, 2) to the Line 3x ~ 4y + 5 = 0. AG, 2) =AG,,y_). Using the formula, jalan tiy tel _|3-9-4-248] 124, le +0 Bre a Avalytic Analysis of Lines and Citler EXNGN GY the aisance tom A(12, 5) tothe ime Sx —12y + Sk = 01s ten unt, Find the possible values oth, Check Yourself 22 1, Find the distance from the pomt P(-2, 3) to the line 3x + 4y +9) 2. Find the distance from the pomt A(1, 4) to the line y = 3x -4 3. The distance between the point P(k, 3) and the line 4x ~ 3y + 5 is 4 units. Find k. Answers > 13 20 setae 5 (46 C. DISTANCE BETWEEN TWO PARALLEL LINES Let dax thy +¢,=0 4: a,x + by + ¢, = 0 be two parallel lines. since d, Ild,, we can wnte © ; ke. soa, Now, let's substitute these values into a, and b, = kb. hax +h by + 0,=0 hay + by + ko, some get dar tba +S = 0. When we compare d, with d., we see that their difference is a constant number. In general, we ean wnite two parallel ines d, and d, as: azar + by +0,=0 ds ax + by + Cy = 0. distance between two parallel fines Let dy: axt by +c, = Oand dy: ax + by + ¢, = 0 be two parallel les. Then the distance between d and d. 1s ithe Appcations n = Ear 90 Proof Emm 91 = Solution The distance of any point A(x, y) on line @, t0 the line dais lax +by te. | fare In the equation of d,: ax + by +0 =0 ax + by =-c,, and so It as amportant to notice that to find the distance between two parallel lines, first of all we need to equalize the coefficients of x and y. Find the distance between the parallel lines x ~ 2y + 9 = 0 and 3x ~ Gy +9 = 0. seartere) 3x-6y+19=0 , was mulaphed by 3. dy: 3x~6y 49 = 3x-6yt9=0 Now, we have ¢, Find the distance between the parallel lines 3x ~2y +9 = Oand 3x + 2y +8 =0. dy; 3x-2y45=0 | 4 3-248 =0) dy: —3n42y48= soe,=5 2 5 } oo elocShe betel 18 gs q=8 Wor Brey 3 ‘There is also another way to solve the problem: az 3x-2y-8=0] The distance between d, and dis the same as the distance of any point on d, or d. to the other ime. For example, A(0, ~4) 18 one point on da, and the distance of A to dis, [3+x,-2-y, 48] _13-0-264)49]_ 13 eee ¥i3. The solution 1s the same. Frey vis vis Avalytic Analysis of Lines and Citler t Check Yourself 23 1, Find the distance between the lines 4x — 3y Oand-12x + 9y + 4=0. 2. The Lines x + 2y + 1 = O and 3x + Gy + k = 0 are parallel and the distance between them as v9, Find k, 3, Find the area of the square whose two sides are on the parallel hnes 2v + y—2 = and av + +6 Answers 1. okey 35 B D. EQUATIONS OF ANGLE BISECTORS Proof Further Appcatons Let dy: ax + by +c, = Oanddy ax + by + 6, ‘equations of the angle bisectors are 0 be two mrersectng lines, then tie Let 1, and [, be angle bisectors of the mrersecting angles of d, and d, as shown im the figure. Let us take a point B(x, y) on l, Since B is a point on the bisector, 11s equidistant to the lines d, and dy, Le. BC = BD. cyan woe need ae inetivvel lexbistel ELUTE YQ Fina the equations of the bisectors of the angles formed by the intersecting ties Solution EX 93 Solution EE 94 = - Qx-y+3=Oand 2x ty-1 Qx-y+3 _ ,2ety-1 Pay aT Qx-yt3=4£Qxty-1) an = ax= 2. 2Qx-yt3=2xt y-1 | Qe yt3=-2e-y 41 and other. Find the equations of the bisectors of the angles formed by the intersecting nes x = 3 and y Bor x— asx a y= Sor yt > x-3=s049) ety 42=0..4 “ wre the equations of the bisectors and notice that they are perpendicular to each In the figure, ABCD 1s a square. A(4, 0) and. D(@, 6) are vertices of the square, Find the equations of the bisectors of the angles formed by the mes AB and AD. D006) AO) Analytic Analyee of Lines and Cites Further Appcatons Draw CH perpendicular to the y-axis. Smee ADOA = ACHD, the coordinates of C are (6, 10). (One of the bisectors of BA and AD 1s AC, because ABCD 1s square and the diagonal is therefore also a bisector of the amterior angle. By using the two-pomt form of ime equation, we can wnte the equation of AC. A(4, 0) and C(6, 10) so, and so 5x-y-20=0. ‘The other bisector is the line passing through A and perpendicular to AC The slope of AC is 3, so the slope of the other bisector is ~+. By using the point-slope form 3 of the Line equation, we can wnite 1 y-0= -2@—4) 5 Sy=-@-4) | xt oy-4=0 In fact, we can find the equations of the bisectors without using the formula. If we find the equations of AB and AD, we can also use the formula to find the equations of the bisectors. Check Yourself 24 |. Find the equations of the bisectors of the angles between the lines 2x -y +3 = O and x4 2y41=0. 2 In the igure, d, and d, umtersect at D. The pots A(-2, 0), BQ, 0), C@, 4), and £(0, 1) are given. Find the equauions of the bisectors of angle D. 3, Im the figure, the equation of Ime d 1s 4x + 3y- 24 = 0. IBF and BE are the bisectors of angle B, find their equations. Answers 1x 3y F2 Bx ty td EXERCISES 1.3 A. Angles Between Two Lines . Find the measure of the angle between the lines 8x + Gy —48 = 0 and 3x —4y + 36 2, The measure of the acute angle between y = 3 and y = mx is 30°, Find m. 3. In the figure, the Lines aoe ty ds Dey ay ax =0 are given, If m(ZAOB) = m(ZBOC), find a 4, Find the measures of the interior angles of the mange whose sides are the kmes x + 9 = 0, x-y+1=0,andy-1=0 B, Distance from a Point to a Line 6, Find the distance from the pomt A@2, 3) to the line 8x + 6y- 19 = 0. 6. The distance between B@, 3) and the Lne 12y~ r= bis © Rind k is 7. The distance from a line with equation y -4 = me + 2) (0 the ongin is 2, Find m. = . 8. The distance between P(1, -2) and the hne 2x —y + k= 018 49 units. Find k 9. The poms A(1, 3), BE2, 1) and C(3, -1) are the vertices of the triangle ABC. Find the length of the alutude of BC. 10, The distance from PC, f to the line 12x + 9y—1= 03 2 umts, Find k C. Distance Betw Lines n Two Paral UL, Find the distance between each pair of parallel Ines. a.-2v + 3y—4 = Oand 2x + 3y-17=0 b.x—y-4=Oand-2x + 2y-7=0 2x + Land 2y = 4y-3 ey 12. The distance between the parallel lines 12x + By -2 = Oand ax + 3y +c = 0 is three units, Find the rano ©, if ¢> 0. 13, Wnite the equations of the Lines which are four © units away from the line 3x + 4y + 10 = 0. 14. The distance between the paral Lnes 3x + 4y ~6 = Oand 4 — ky +4 = 018 p. Find kee. D. Equations of Angle Bisectors 16, Waite the equations of the bisectors of the angles formed by the lines 8x ~ 4y + 10 = 0 and 12 + by-24 = 0. Analytic Analyee of Lines and Cites 16, Wnte the equations of the bisectors of the angles formed by the hes 3y = 2x + 1 and 3x = -2y —4. 17, Find the equauions of the locus of the pomts © which are equidistant to the nes x + 3y—1=0 and 3x-y +2=0 CET ‘18. Fund the tangent of the acute angle between the Lines joining the points (0, 0), @, 3) and 2, -2), Gd). 19, Find the measures of the interior angles of the tangle with vertices A(10, v5 + 2), BO, 2), and, cdo, 2) 20, Find the tangent of the angle between the lines whose x- and y-intercept are (a, -b) and (b, -a), respecte, 21, Find the equations of the two straight lines which ass through the point A¢4, 3) and form an acute ange of 45° with the ine 2x —y +7 = 0 22, Wnite the equation of the line parallel to the line ~3x + 4y — 16 = 0 and whieh 1s at three umts distance to the pomt PCL, 1). 23, Find the coordmates of the pomts on the hne 2 y = 2x whose distance to the line 3x + 4y - 12 = 0 is two units 24, Find the equations of the lines passing through © AQ, 0) which form an ange of 45° with the line etytl 25, Find the lengths of the alutudes of the mange © bounded by the lines xy = 1, 4v + 3y = 4, and xtys Further Appcatons 26, In the figure, the dines d and d, intersect at B m(ZABC) = 45°, AC, hd, BCA, -2), and ©, 0) are given Find ke 27, Im the figure, ax ty =o, Ac, 0), “ and d,|id, are given. IAABC as an equilateral triangle, ‘indats area 28, The lines d, and a. intersect each other at A(0, -2) Find tan ot 29. In the Agure, °F 4B 1 a, the equation of d, 1s y = 3x, and the equanon of d, 15 xy =O. TEAB = 22, find the area of AAOB. SYMMETRY ) 1G: Ina plane, two figures or objects which are equidistant to a fixed reference pomt or line are called symmetne figures. In a football or basketball field, if we choose the line m the mid-field as a reference lime, the other lines im each half of the field are the symmetry of one another with respect to Uhis Line. On the number lune, positive numbers and negative numbers are the symmetry of one anoth- er with respect (0 the origin. For instance, 5 and -8; 10 and -10; /3 and -V are symmemic ‘wath respect to zero on the number line. An object and its amage in a flat murror ase te symmetry of each other wath respect to the surface of the murror A. SYMMETRY OF A POINT ES 95 Solution 1. Symmetry of a Point with Respect to the Coordinate Axes Let us find the symmetry of point A(x, y,) with respect to the axes. The symmetry of the pomt Aig ug) with respect to the ¥-aXIS is Ay(%y. =)g) ‘The symmetry of the sume pomt with respect to the y-axis 18 AX-%y $ 4) with respect to the axes. Find the symmetry of the point ( 34. The symmetry of P(-2, 4) with respect to the > Dt ansis ROS, 2 issrmmety wth emer tothe eames 2,4 Gialehe Aastea af tteerenl le EE 96 Symmetry Check Yourself 25 1. Ifthe symmeuy of the pomnt A(4, —1) with respect to the x-axis is B, and 1ts symmetry with, respect to the y-axis 18 C, find the area of AABC 2. The svmmetry of which pomt with respect to the x-axisis the same as the symmetry of P(-2, 3) with respect to the y-axis? Answers 1.8 square umts es Parallel to 2. Symmetry of a Point with Respect to the Axes Let A, and A,be symmetries of the point A(t, 1.) with respect to the lines x = a and y = b, respectively. Let us find their coordinates. In symmetry with respect to x = a, the ordinate of Ag, YJ.) Temains unchanged. In symmetry with respect to y = b, the abscissa of A(X, yy) temas unchanged, IfA,(%,, 9,) 1s the symmetric point with respect to x =a then “tm x)= 24-2 | = AQa-a,.4,) w= % TEA,(a., y,) 1s the symmetnie point with respect to y = b then qi y= 2b- 9, thi, = AG, 2b—19)- 2 mam Consequently, the symmetry of A(ty, 14) with respect f0x = @18 A,(2a ~ xy 19), and its symmetry with respect (0 y = bis Ay(%y, 2b ~ 99) Find the symmetry of A(-3, 2) wath respect to each line, ax=4 bys Solution a, A,Q,.¥)) = A,@a- xy 4) b. Ad(te 12) = AG’, 2b 90) Using a = 4 and x, Ustng b = 5 and x, =-3, y, = 2, A,Q-4 +3, 2) =A,(1, 2) A,3, 2-5-2) = ALCS, 8). Check Yourself 26 1. Find the symmetry of A(4, -3) vath respect tox = dandy = -2, respectively = 2. Find the symmetry of K-2, ~4) with respect to x = 2 and y = 1, respectively, and show them on the coordinate plane. Answers 1AC8,-3, AAD) 2.484), 4,2, 6) 3. Symmetry of a Point with Respect to Another Point Let us find the symmetry of the pomt A(x, 9,) with respect to the pomt B(a, b). If A(x, 9) 18 the symmetry of A with respect to B, then AB = A,B and B(a, b) is the midpoint of AA,, so the coordimates of B are atx and b= UM Atay) a This gvesx=2a-x, and y=2b-y.20 a,b) A, 1) = A,Qa =x, 2b = 99 1fB(a, b) = O(0, 0), the symmetnie point 1s Aly -) Consequently the symmetry of A(ty 9) with respect to Bea, b) 18 A\Qa ~ %y, BH ~ yp, and as symmetry with respect to the origin O(0, ®) 15 Ax» -u9) Acta) Find the symmetry of AS, 4) with respect to each pomt. a. BU, -3) b. the ongin a, Let A,(x, y) be the symmetric pomt, then Ale I= ACAD om B(a, b) = BU, -3) y=2b—1, so x=2-1-(9)=2+9=7 y =2-(3)-4 = 6-4 = -10, Therefore, the symmetnie pomt is A\(7, -10). b. The symmetry of A(-9, 4) with respect to 0(0, 0) 15 AW-(-3), -4) = AG, 4). Check Yourself 27 1. Find the symmetry of A(4, -2) with respect to BG, 1) and to the onigm. 2 The syammetry of PCL, k) with respect to R¢-2, 3) 1s on the he 2x —y + 2 = 0. Find k 3, The symmetry of PCL, -2) with respect to A(a, 2a ~ 1) is on the line 3x + 4y +2 = 0. Finda. Answers 1.A,Q, 4), AC4,2) 2.14 Avalytic Analysis of Lines and Citler ES 98 Symmetry Solu 4. Symmetry of a Point with Respect to the Lines y =x and y=-« In the coordinate plane, the symmetry of a pont A(x,, ¥,) with respect (0 y =a 15 By X)), and ts symmetry with, respect to y = -¥ 8 Cy, -¥,). Let us think why. In the ‘igure, since BF = AF and OF 1. AB, AOAB 1 an wsosceles, ‘mangle and OA = OB. m(ZEOP) = m(ZFOH) = 45° and m(ZBOF) = m(ZFOA) then m(ZEOB) = m(ZAOH) and also m(ZE) = m(ZH) = 90°. Hence, AEOB = AHOA and OH = OE AH = EB » Consequenty, the symmetry of A(ty 1) with respect to = % 35 Bly Ny) ‘Sumulanhy, the symmetry of A(ry. y) with respect to y = +x can be found as Cy, ‘The symmetry of the point P(-2, 1) with respect to y = respect to y = x isC. IfC is om the line 2x ~y + k x 1s Band the symmetry of B with find ‘The symmetry of P(-2, 1) with respect to y = with respect to y = xis C2, -1) CQ, 1) as on the Line 2x —y + k= 2.2-C1) +k= 0,50 21s B(-1, 2) and the symmetry of BC-1, 2) , sot must satisfy the equation. Check Yourself 28 1. Find the symmetry of 7-4, 7) with respect to the given lines, aytx=0 by-x=0 2. The symmetry of P(e, ~7) with respect to y = —x is the same as the symmetry of R(k— 1, m) with respect toy =x. Find k + m 3, The symmetry of the point AQ, 5) with respect to y = —v is B, and its symmetry to y 4s C. Find the tangent of the acute angle between AB and BC. Answers a ad 2 7 ‘= En 99 Solution — 5. Symmetry of a Point with Respect to a Line We know how to find the symmetry of a pomt with respect to the Imes y = v and y = x, How can we find the symmetry of a pomt with respect to a different Ime? Let us look at an example. ‘As shown in the figure, the symmetzy of pout A(t, ,) wath respect toalwne d: ax + by + c= Os the same as the symmetry of A(x, -) ‘with respect to the point H. Therefore, we need to find the coordinates of the point H. ‘We can find the coordinates of Hm tree steps land m,, ==). 2. The equation of the line AA, 1s y~ a = My,» @—%)- 3. Point His the intersection point ofthe Imes AA, and d, so we ean find its cooxdinates by simultaneous solution ofthe equations. 1. AA, Ld, 50 my, +m, Now we have the coordinates of the point H. we ean take the symmetry of point A(t, 1) with respect to H. This is the symmetry of A(x,, y,) with respect to the line Find the symmeny of A(-2, 4) with respect to the ne x ~ y ~6 = 0. Let A,(a, b) be the symmetnie pomt. AA, Ld and m,=1 ‘my, mM, = —1 and my, = -1. The equation of the hne passing through A(2, 4) with the slope m = -Lis y -4 = -1@ + 2), so the equation of d sy + x-2= 0 dnd, = (K} and to find the coordmates of K, let us salve the system of lear equations of d and d, x-y=0 y } = Ken )=K4,-2) ety=2 Now the point K(4, -2) is the midpomt of AA, where A(-2, 4) and A,(a, b) are the endpomts. Rea, 2) = 28%, 844), soa-= 10 and b= Band the symmetne poms 4,(10, -8) 2 Check Yourself 29 1. Find the symmety of the point AC. 2) with respect to the line 2x + y~4 = 0. 2, Find the symmetry of AQ, -) with respect to y = x ~ 2. 3, Find the symmetry of PCA, 1) with respect to 4x ~ By ~ 1 = 0 Answers LQ,2 2€10) 3 4) Avalytic Analysis of Lines and Citler B. SYMMETRY OF A LINE (OPTIONAL) Solution Solution 1 1. Symmetry of a Line with Respect to a Point Let us-‘find the symmetry of a hme a: ax + by + ¢ = 0 with respect to a point Bp, @. The symmetnc line, say d,, 18 parallel 0 d: ax + by +e =0. Now, let P(x, 3) bea pomt on the ine @,, $0 the symmetry of Px, 1) with respect to B(p, 4) 18 P,Op —x, 24 - 9) Since P, is on d, the coordinates of P,(2p — x, 2g ~y) satisfy the equation d: ax + by +¢= 0. ‘Therefore, a(2p - x) + Bq y) + = 01s the equation of the line which is the symmetry of ‘with respect to the point B(p, 4) We can see that we just substitute the symmetry of P(x, y) in the original Line equation and. ‘we get the equation of the symmetuic line aparhby te =O dea shy Find the symmetry of the line -2x + 3y + 5 = 0 with respect to BCA, 3) Substitute the symmetry of P(x, u) with respect to B(-4, 3): the equation of Line, PQ: (4) —x, 2-3 ~y) = P\-8 — x, 6 —y) satisfies the equation of the line, 2x + 3y +9=0 2. (B=2) +3-(-s) +5=0 16 + 2x + 18-3y +9=0 2n —3y + 39 = 0 is the equation of the symmetric line. Find the symmetry of the line 2x + 3y ~ 6 = 0 with respect to P(3, 1) Let the symmetry of a point B on line d, with ae +3y 6 =0 respect to P(3, 1) be a pomt A on d, The symmetry of B(x, y) with respect to P@, 1) 18, AG ~ x, 2-y).A is on d, and its coordinates satisfy the equation Qe + 3y-6 =0 2 ©-x) +3-Q-y)-6 =0 12-2r + 6-3y-6 =0 2x -3y +12 =0, So 2x + 3y — 12 = 0 the equation of d, ais.) Symmecry Solution 2 The symmetry of the lme d,: 2x + 3-6 = 0 with respect to P(G, 1) is the line d.; 2x + 3y +p = 0, and the line t: 2x + 3y + k = 0 passes through PG, 1) and parallel to d, and dy Smce P@, 1) son 2-343-1tk k=-9 and b:2x + 3y-9 Now d, || fand d. ||! and PA = PB, so by using the distance formula between two parallel hes, we can wnite 1-6-kl _ Ip-kL , 1-6 +91_ 1p +91 VB Vis -6, Therefore, the equation of d, is 2x + 3y ~12 = 0. Let the symmetry of a line ax + by + c = 0 with respect to a point Pom, n) be the line 4 ax + by + p = 0. Aline passmg through P and parallel to d, and d, is: ax + by +h Since P is at the same distance to the lines d, and d,, the distances between / and the bnes 4, and d, are equal, Hence, the constant term of Us the arithmetic mean of the constant terms of d, and d. dzaxtbyte baxtbyt deaxtby +p Note The symmetry of ax + by +c = 0 with respect to the ongin 1s ax ~ by + ¢ = 0. Check Yourself 30 1. Find the symmeuy of the line 4x + y ~4 = 0 wath respect to P(-1, 2) 2, Find the symmetry of the Line 4x ~ 3y + 4 = 0 with respect to PQ, 3), 3. Find the symmetry of the ne x + 3-2 = 0 with respect to the onigm. Answers Lae ty +8=0 2-dv4 ay 42 Bx t3y+2=0 tte Anais of Line ad Cicer Solution 2. Symmetry of a Line with Respect to Another Line a, Symmetry with respect to a parallel line Let d and Ibe two parallel lines, then the symmetry of d senatorial Lan i a AL aikit Let their equations be Se Ee b dav + by +o=0 ie Reasehy en 8 Farspea it} Lax + by + p=0. Since the line ke 1s equidistant to the Les @ and f, +P 2 ‘and p=2m~c. By subsotuting 2m ~ can the equation of l wwe get the equation of the symmetric line Lax + by + @m-0) = Find the symmetry of the Line d: 2x + 3y + 4 = 0 with respect to the Line k: 2x + 3y @aehoved=o) 28 aay fe De-+3y - 273 - Let the symmetry of d with respect to k be [: 2x + 3y + p = 0, then since k is between @ and l, are, a 1, $0 p=-GandsoL2x+3y ~ Check Yourself 31 1, Find the svmmetzy of the Line 2v + Sy - 2= 0 with respect to the Line 2x + Sy +3 = 0. Answers 1 e+ Sy +8 = b. Symmetry with respect to particular lines ‘To fmd the symmetry of a line with respect to the Les y = x, y = =x, the x-axis, the y-axis, and the hnes x = m and y = n, we cam use the same idea of the symmetry of a point wath respect to these lines. We find Uat the symmetries of a pont A(x, y) with respect to the Lines above are (a, x), Cu, -»), (%, -u), (ey), Qm —x, 9), and (x, 2n ~y), respectively. : = Ens 104 Solution Solution Solution ‘We can use the same pattern to find the symmeuy of a line The symmetry of the Lne ax + by + ¢ = 0 with respect 10 is e+ ay +o=0. 2. is -br-ay += 0. 3. the x-axis 15 ax —by +e =0. 4. the y-aus 1s -ax + by be =0. S.x=m As am—x) + by +e =0. By=n as ax tb@n-y) +e=0. Find the symmetry of the ine 3x ~ 4y + 1 = O with respect to the hnesy =x and y ‘The symmetry with respect to y = xis Ax + 3y +1 is 4x—3y +1=0. The symmetry with respect to y Find the yammetry of x -3y +4 ‘yath respect to the axes. The symmetry with respect to the x-axis 18.x + 3y + 4 = 0. ‘The symmetry with respect to the y-axis is —x ~ 3y + 4 ‘The symmeny of a line d with respect to the x-axis1s kx ~3y + 1 (othe y-asis 2x + ky +4 =0.Find k, and its symmetry with respect Let ax + by + ¢ = O be the equation of d. In the symmetry with respect to the x-axis, the sign of b changes: with respect to the y-axis, the sign of @ changes. Consequently, the slopes of the symmetric lanes are equal, and the lines are parallel. fear Sy + 1 =O and -2x + hy + 4 = O are parallel Lines, so, Check Yourself 32 |. Find the symmetry of the ne 2x + y ~2 = 0 with respect to the line x 2. The symmetry of a line y = mx + 2 with respect to the y-axis mtersects the x-axis at 4@.0) nam 5, nthe coxtnats arte merseevon pot ethe sme athe x + 29-1 =0 with pet ois and the srmmety of he sme ine withers fo = 4. The metry of aie 2x hye O wih eset thee y = = Othe ne x +3y—4= 0. Find O45 Answers 12x ty-10 zi 5 tte Anais of Line ad Cicer c. Symmetry with respect to an intersecting I Let the line sax + by + c= Obethe symmetty ofthe line d: a,x + by + ¢, = 0 with respect to line e Regeiiy Hae TA A pera To find the equation off, follow the steps: Sef 1. Find the coordinates ofa point A, dk = {A} 2. PH = HN and m(ZPAH) = m(ZHAN) 5. We know the slopes of d and k, so we can find fan et and then usmg tan a, we can find the slope ofl 4 We can write the equation ofl using the formula y ~ of Land A(x, y) 18 mtersection pomt of the Imes, RNa taxthy+ee0 r(x x,), where m is the slope FEZ TOG ne the symmetry ofthe tine x + y—2 = Owith respect theline x ~2y + 1 = 0 Solution 2+@+y-2=0) x-2y +1 =0 3x 0,80 x = Land therefore y = 1 and so the intersection point 1s AQ, 1) The slope of d = m, the slope of if ban + by + tane, = uno, 2m, 1-2m, =-6-3m,, sothe dope of [3s m, Las the ne passing through A(J, 1) wath slope m, = ~7, so its equation is, 16-1) 0. y- wey Check Yourself 33 1. Find the symmetry of 2x + y + 2 = 0 with respect tox + 3y-1= 0. 2. Find the symmetry of 3x + y—1 ‘wath respect tow —y +1 = 0. Answers Lx-2y43=0 2x 3y+3=0 EXERCISES 1.4 A. Symmetry of a Point . Find the symmetry of A(a ~ b, -b) with respect to a. the x-axis, > the y-axis. 2. The symmetry of B(-4, -) with respect to the x-axis is K, and ats symmetry wath respect to the y-axis is L. Find the slope of KL 3. The symmetry of L(a, b) with respect to the x-axis 1s the same as the symmetry of T(-10, 9) with respect to the y-axis, Find a - b 4, Find the symmetry of M(-12, 7) with respect to each line, ax=-3 5. The symmete pomt of N(a, 7) with respect to D(-2, 6) has 29 units distance from the origm, Find possible values of a 6, Find the symmetry of L(-7, 4) with respect to = =— ag by= ‘J. The symmetry of the pomt A(m, 3) with respect to the y-ausis on the hne 3x —y + 4= 0. Find m, 8. Find the symmeuy of P(1, 3) with respect to x4 2y-1=0. @ The symmetry of A(2, -1) with respect to y= my #15 (4,9), Find m +m. 10, The symmetry of P(-3, 1) with respect to 4x ~3y —5 = 0 asK. Find PK. LL, Find the symmetry of P(, -1) with respect to ysx-2 B, Symmetry of a Line (Optional) 12, Find the symmetry of 3x + y +4 = 0 wath respect to. each pomt. a. the origin b. PC, 1) tte Anais of Line ad Cicer 13, Find the symmeny of 3x + 2y - 3 = 0 with respect to a. the x-ans. b. the p-axas 14, Find the summetry of y = 2x + 1 yath respect to the line x = 3. 15, Find the symmetry of 2x 4y + 1 =0 with respect (0 each line ax by 16. Find the symmetry of 3x + 4y + 5 = 0 with respect to the line 3x + 4y - 1 = 0. 7, Find the symmetry of x + 2y — 3 = 0 with respect to the ine 3x + y + 1 = 0. 18, The nes (2m —1)x + Qu + 3)y +5 = 0 and © oy —@— Dy + 9 = 0 are symmetric with respect to the x-axis. Find (m,n) 1g, In the coordinate plane, the symmetry of © q: 2x —y + 2 = 0 with respect to the line It is Lx + 3y + 1 = 0. Find the slope of k. 20. The symmetry of 3 + 24 —2 = 0 with respect to (1, 2) passes through R, hk). Find k, Cit maou QL. The symmetry of P(S, -2) with respect to y = x15. A, and its symmetry with respect to the ne y = ws B. Find the area of AAPB 22, The symmetry of P(3, ~4) with respect to A, 2) as K, to and ats symmetry with respect to the Line y = xos L. Find KL, 23. The symmetry of 2x — 3 + 2 = O with respect to. the ne y = x passes through A(@, m). Find m. 24, Find the equation of the locus of the pomts 8 which are symmetries of A(k — 1, k + 2) with respect to Bk, k— 1), (k= B). 25, Write the equation of the lime which passes © uyough the symmetry of PCL, 3) with respect to the ine 3x + y — 4 = 0, and which as perpendicular to the ine x + 2y + 2=0, 28. The symmetry of PCL, 3) with respect to © Age, y) is on xy + 1 = 0. Find the equation of the locus of A Symmetry “= Cr a ‘Tnow isa one-to-one correspondence between the points ina plane and the Cartesian coordinates. The points can bbe represented by two components, the abscissa and the ‘ordinate, A(t, y) ‘The distance between the points A(xy y,) and Bly 16 AB =f, = +0, ¥ ‘The coordinates of the midpoint of a line segment AB wath endpaints A(x, y,) and B(x, ys) are and | == Give 2 le segment AB wi Al, 3.) and BC, 39, Cte, 4s the pamt mach cides AB nthe rao cA o IEC dindes AP internally, Givena triangle ABC with vertices A(t, ah), BGts 2), 06 + Cees the cannes GEER, we and AABC = © 1X(Ue-4) + 04-1) +80, - | ‘The angle between a line and the positive cireotion of s-axis mansured sn a counter-clockwise direction 1s ‘alled the inclination of the Tine If eis the inclination of Line, the tangent of ais called the slope ofthe line, m = tance ‘The slope of a hne passing through the points A(x, ¥,) wom and Boxy y.) is m 18d, 1d, then m, ied, Ld. then mm, ‘The equation ofa line which passes through the point Ace, y) wita slope me y—y, = m= (c-X,) — ‘We can wnie the equation of ime mn two diferent formes: Lax+by+e=0,abceR 2 y = mx +n, m, me R, mie the slope of the line ‘The equation of a line passing through two fixed points ACK UD and Boa, 4,38 ew arn ‘The equation of a line wath known X- and yeuntercept is where @ 35 the Xuntercept and b is the ‘yeinteroept of the line y = Os the aquation of the X-ais X= Os the equation of the y-axis ‘The slope of aime given im the form of ax + by + 5 Given evo lines dy ax + by +c, dy ax + by +e aie i then the lines are comeident yb & 2 if S244 % then the ines a pale Bs 1.2% tem the tines intersect at one point aby Bit The equation ax + by +e + Re (aa + by + repmsents the equation ofa bunch of lines ‘The regions of the coordinate plane separated by a line ay + by + ¢ = 0 am defined by the inequaliues ax + by +e 0, ‘The tangent of an acute angle between the lines my My y=mx +nandy =mx+n, i anes ‘The distance from a point A(x, y)) toa line dart byto=01 toy, +e 1 = Analytic Analyee of Lines and Cites © The distance between two parallel lines aos ty + ox and dy ax + ba +6, Iq=e Wear © The equations of the angle bisectors of the Lines ap ae + by + and dy a + © The symmony of a paint A(t ye) with respect to 1 point B(a, b) is A\(2a — x5 2b — a). the a-axis 36 AG - ys). the y-anis 6 AKy Ye) alme x= 46A(Qa-a,, 9) aline y= bis Aa, 2— yo) the Ine y = #15 A,(Yy #0) the ine y = 15 Aso -¥9) The symmetry of alin ax + by + © math espect to 1 a pomt B¢p, @) 16 a(2p— x) + (29 -y) +6 2 aline ar + by + m= Oisar +by + (Om —0) 3. the Ime y = xisby + ay +¢=0. 4 tha Ine y = -ris—br—ay + 6 8 0 the anes ax -by +.¢=0 the y-anis is ax + by + 0 = 0 aline x =m isa(2m—x) + by +e=0. aline Ine y =n isax + bQn—y) +e Centos 1. What is the coordinate plane? 2. How can a point be represented in the coordinate plane? 3 Define the concept of line. Find examples from daily ie roillustrate the concept of line 4 Gwen the coomimates of the vertices ofa rectangle, hove ean you ealeulate us area? 5 Explain the meaning of the terms inclination and slope of aline 8 What is the wlation between the slopes of two parallel ines? What is the relation between the lines y y=mx+m © Beplain why the line x = 51s parallel to the y-ads 9 What are the possibilities for the relative position of two ines? 10 How cam we determine that the angle between two lines san acute ange? 11 How can we determine that two lines ate parallel to one another? 12 What is the procedure to find the area of 2 square whose two sides axe on parallel ines? 13 Desenbe the procedune to find the equations of the angle lsectors 14 Desonbe the concept of symmetry and give some examples of symmerry m dasly fe 15 Gre examples of symmeme Zgures with respect toa kne anda point 15 Desenbe the procedure to find the symmeny of a point wath respect toa Line mx and Hey David! ‘What are you doing? Don't waste time! ‘Tomorrow we've got an analytic geometry exam on fines! We need to study! No problem! Tve already been online for three hours. Analytic Analyse of Lives 1, Whatis the length of the median passing through the vertex A ofa tnangle ABC with vertices A(4, 7), BCA, 2), and C@, 4)? AS B86 C)7? Ds FIO 2, Ifthe pots A-2, -1), BQ, b), and C4, 8) are on the same Line, what is k? Al B2 ©)3 DA Ed 3, What is p. if the slope of the line 2px + (p-1y + 1 = 018 a? pk ot p- wt 1 1 3 z 4. The centroid of ABC with vertices AC2, 1), BQ, 5), and CG, -1) 5 on the line 2x ty + k-1=0, Find k 1 5 8 10 Aa B- O-2 D-H yoy O-7 D> By 5, Find tan oan the figure. 6 6 3 2 as 2S of pt < 7 Bs Sig DS 6. The Lines x + 3y -8 = 0 and ky + y -6 intersect on the Line 2x — y ~ 2= 0. Find k. AL B2 C-2 Ds BB 7. The distance of Pt, 1) to the ne x —2y + 2 = 01s 245, What is the sum of the possible values of k? AO 81 ©2 D1 H-2® 8, Aand B are the x- and y-intercept of the Lime Sy + 12y - 60 = 0, respectively. Find AB A BIS OM DI 2 Armytic Amy of Lines aad Cntr 9, The angle between mx - y + 2 = 0 and 2x + y- 1 = 0 is 45°, What i a possible value of AO B)-1 2 D2 HB 10, ACA, 2) and B4, 3) are given. C 1s on the hne ac _ 3 segment AB and AC AB 3 What is the sum of the coordinates of C? AL B-1 ©2 D2 BB Ill, Which of the following lines has the greatest snehination? A) Qe-y + 1=0 Byx + Qy-1 Cyy=srt4 Dy = Bx Eyyares 12. In the Neure, , e AOBC 1s a rectange. ; ft ays y = 2eand dy y=, and t us, ay A(O, 8) are gven. 6 ‘What is the area of COD? A688 C12 D)1s B18 (Cheer Rayo Tt A 13, The symmetry of (k - Dx + 2y - 6 = 0 with respect toy =x passes through AQ, 1) Fmd k. Al B2 O88 D4 E-4 14. The figure shows the graph of the height of a plant yersus time in years, What is the height of the plant im the eighth year? y lengthy wear Ayo B10 C12) SF) 20. 15. Find the equation of the line which passes through the common pomt of the bunch nes (e+ Dx- Ok + Dy +h-2=0, and which 1s parallel to the hne x + 3y — 1. Ayxt 3y414=0 Bax ty ) 3ety- B)xt3y-4=0 Ce +ay43=0 1G, The points A(2, 1), B(2, 3), CG, 4), and Dee, 1) are given. If AC LBD, what 1s k? A4 BB O-2 D-l 4 ‘= 1, The points Pom, u) and Rx, y) are on the Ime 4x -3y-1 = 0. Fmd B= ay 1 4 a 3 a 3 Get & %3 DG >, DZ Pl 2, The pomt A(a - b, a ~b) 1s m the second quadrant of the coordinate plane. In which quadrant 1s BO'a, bay? Ar Bo om DV 1) at the ongin 3. AGI, 3) and B(4,~2) are ven, Find the equation of the line perpendicular to AB which passes through the ongin o Byx ty Ayxey C)y =2x D)y= = By=5 4, Whats the abscissa of the pomt which is on the Line y = -2x + Sand equidistant from the points AGH, 2) and BU, 4)? Al Ba O2 D2 HB =: 5, Which equation defines the locus of points which are equidistant to the lines x + 3y - 1 = O and Bx-y +4507 Ay4x ty-1=0 ©) ax + 2y +3 =0 P/Q + 3y + 1=0 3B) Qe -4y +4=0 D)xt2y+4=0 6. In the figure, ACOD 1s an equilateral tangle, A@COD) = 123 and m(ZCAD) = 30° ‘What isthe equation of AB? Aya ar +6 Se C)y = Bx + 83 Dy Ey =ar4e8 7. In the figure, P(O, k) as * ‘moving along the y-axis. Bas) AG, 1) and BG, 5) are ven. Which value of ke gives the smallest value of the sum PB + PAD (0,4) »% yi 58 y® y® 8. The symmetry of a pomt on the Ime ax + 3y + 1 = 0 with respect to the ne x ty = 01sAQ,-1), Find a, As B44 OF DBI HA Armytic Amy of Lines aad Cntr Q.A and B are the x and y-intercept of the Ime ¥ + 2y ~ 12 = 0, respectively, What a5. the equation of the median of the hypotenuse of the rangle formed by A, B, and the origin? Ayx + 2y=0 B)x-Wy =0 ory Dax ty =0 E)y = 3x 10. In the fgure, ¥ CH 1 ABis given. What is the length of CH? Al0 B86 SOYA 11, AG, 2) and BG, 4) are the endpoimts of AB. The nes d: 3x + 4y ~ 12 = 0 and AB intersect at a point P, What the ato 227 1 1 Yo PE OD DS YH 12, The side BC of the square ABCD 1s on the Ime 3x ~4y + 2 = 0. Given AQ, -3), find the area of, ABCD. Aj64 8/49 C32) 2B) IB Chapter Rau Te 1B 13, Two of the vernces of AABC are B(-3, 3), CC, 9), and Ais the common point of the bunch, of ines with equation (im Dx + (m+ QHy—m +4 of AABC. 0. Find the area, Al BS 2 Ds 4 14, The symmetry of PG, 9) with respect to Rk 1, 4) 8 on the y-axis. Find k 8 . 8 ‘ XZ B83 Of DF Hs 15. AQ, 5), BG, 1}, and P(a, 0) are given. Which value of @ gives the maximum value of PA ~ PB? 8 . uw a4 BF OF DF we In the figure, CDEF is a square. d: y = x passes through C, and d, passes through A(-3, 0) and, BQ, 2). What is the area of the square? Ay16 8)27 C82) BBE) AD = I. Ina line d: ax + by + ¢ = 0,a,b, ¢ are integers such that a ¢ b+ c, The area of the tangle formed by the coordinate axes and the line dis =~. mind the value of Dab 5 B44 O83 DF Hr In the figure, KL 1 LB, and KL = LB. Given d: 2x ~3y +30 = 0, find the area of AKLB. A) 426 By 226) 3413.) 2VI3_E) 52 3, The points ACL, -1), B(@, b), and C(-9, 8) are AC _3 ven. A, B, Care collinear and 5E= 5. Find the coordinates of B a3.) B) €3,3) ©) 4-3) Das » as) 4 What is the abscissa of the closest pomt of the Line x ~ 2y + 1 = 0 t0 the pomt AU, 2)? ot of of D+ me fos = 5. The pomts A(2, 3) and B(L, -2) are on different sides of the hne 3x ~ y + k = 0. How many possible integer values can ke take? Al B82 C8 D4 HS 6, Find the equation of the line parallel to the line 4x — Sy + 6 = 0 which amtersects the Line y-x-3=0on the y-ans, A) oy ~ 4 2B) Qya ee ©) yx F9=0 D) Sy— 4x —15 BF) by-4v + 15=0 7. Which graph shows the solution set of the system yer), ys-x+2]" Ayoy B) . y ah, lo (0 * Cy D) # 4 2 ‘ SS ale yo # Armytic Amy of Lines aad Cntr 8, Find the area of the tangle formed by the lines x + 2y-4 = 0, 3x + 2y- 12 = 0, and the y-axis, A)B BIZ C) 16D) 24) BD. 9. The symmetry of the origin wath respect to the Ine x+y —k = 018, Given OP = 6V2, find k As B44 OF DE HO U0. The equation of dy mm the figure is, rty-6=0 OABC 1s a square and OA = 2 ums. Given BH L KL, whats BH? Al B) OF DB Hw a> 0, b> 0. given. The area of the inane Formed by the ne ax + by +2 = O and the io coordinate axes s four square units. Find a YN rom this, we get Ca, ) = C(3, 0 ’) and r= 4 units. = ii i pti at te 5 Determine the center and radi ofthe circle x* + (y +2)" = 7, and sketch ts graph, Solution We arrange the given equation in the form @-0) + @- Cay? =f Fie Thus, we get Cla ) = C(0,-2). and ha i r= Ji unas, Check Yourself 1 1, Write the equations of the circles with the centers and radii given below. 2C@1, r=2 Ac&-p, r=3 b.c@2,0), r= 8 ©. €(4,-4), at 60,0, rz £008, 8), r= JO 2, Find the center nd the radius of the etele defined by each equation, and stetoh as graph. axtgyts3 cea tot B= > bt eOtD=9 A@tayrey=2 CSET) G fine the equation ofthe circle whose diameter is AB with endpoints AC-2, 6) and B(, 12) Solution Since AB is the diameter, the midpoint of AB is the center of the enrele and CA = Cl “349 6412 So the center C(a, b) = CC 7 = CU, 9), and the radius r=CA= (3-1) +(6-9y = floT9 = So the equation as (x — 1) + (y~ 9)" = 26. Ac, 0) uunuts, [HESTUIES 7 vetermme me equation of the cucte with center CC, -3) which passes through the pout PC-2, 1). Solution Since the point P-2, 1) son the circle, r= CP = Q=CByHCO—D® = 8 unis Now we can write the eqution ofthe circle with the center C1, -8) and radius r = 6 unis as @— 1) + +3)" = 25. Equation of «Circe 101 = (MESTOTINSE G wre the equanon ofthe acl centered at (2 ~8) which is tangent tothe Ime dv ~3y +3 = 0 Solution The distance from the center to the tangent line 1s equal to the radius. Hence, by the formula of the distance of a point to a line, Thus, the equations 2)" + Y + 3 = 16. Find the equation of the circle passing through the pomts AC-2, 3) and BG3, 4) whose Solu center is on the Ime y = 2x. Let C(a, b) be the center of the circle, Since the center 1s on the line y = 2x, we can write b = 2a, and Ca, b) = C(@, 2a) On the other hand, =CB r= a+ 2+ @a-3y = fara ra + Aa 44+ 4g"—120 +9 = 9" + 6049+ 40° 160416 2a = 12 a=6. Now let us find the radius, BC= r= fat3) +Qa—4y = (a, 2a) = C(6,12) (x 6)" + (y-12)" = 149 is the equation of the circle. as iyi Arya of Lies ard Ctr (EEQIEE 10 Fin the equation of the exrcle which is tangent to the lines y +1 = 0 and y 5 = 0, and Solution Equation of e Cirle ‘whose center 1s on the Lne 2x - y = 4 Smice the cucle 15 tangent to the Lines y = —1 and y = 5, the center will be on y = 2 which 1s equidistant from these lines, ‘Smce the simultaneous solution of the line equations 2u ~ y = 4 and y = 21s (3, 2), the cemter of the circle 15003, 2). o-Cl)_6 2 2 So @ ~ 3)" + (y ~ 2)" = 9 is the equation of the circle. The radius r= 3 umts. Check Yourself 2 |. Write the equations of the eircles passing through the point P with the center C given below, a. C0, 0), P(-3, 4) b. CO, -2), Pa, 8) ©. CG, 0), P(l, 4) 4. CG, 4), PO, 0) 2. Find the equations of the circles with center C which are tangent to the given lines, 4 CQ, 0), dx 12y + 10=0 b.C@, -3), 3x — ay +12 =0 3. Find the equation of the etrele with center on the Lme 24 ~x + 2 = 0 which is tangent to the lines y + 1 = Oand y-3 = 0. 4, Wnite the equation of the circle with center on the line 4y + x ~ 8 = 0 and with scmtercepts at A(I, 0), BG, 0) Answers 3. @ =a + @=1) 44 (ay tye a. Equation of a circle tangent to the 2-axis ‘The radius of a circle which 1s tangent to the x-axis with center C(a, b) is = |b], and its equation is, (a8 + y= SEMEN WY rina tne equation of the eile which is tangent to the x-axis and whose center is C(3, 4) jon The eicle as tangent to the x-axis, Wh r= bal @- 3 + +4) and the equation is b. Equation of a circle tangent to the y-axis The radius of a circle which is tangent to the y-axis with center C(@, b) is r= Ja, and its equation as ay + W-BE= at TQ Ana ine equation of the cite which is tangent to the y-axis and whose center is C3. Dy 7 Solution The excle is tangent to the y-aus, sor = al,r = 1-3] =3 units, and the equation is (x + 3)" + (—1)"= 3% = i titi at te c. Equation of a circle tangent to both axes The radius ofa circle which 1s tangent to both axes with, center C(a, b) is r= [al = |b] Hence, the possible equation for such a circle is as follows: an the first quadrant (x - )?4+ @ —*= 9 1m the second quadrant (x + N® + (y—NP= 18 in the Uird quadrant («+ N84 GW +7) in the fourth quadrant (@—-)® + &y + N¥= rh In all cases, the center of the circle is on the line y = x or y = White the equation of the circle which is tangent to the coordinate axes with center CQ, 2 Since the circle is tangent to the axes, r = |a| = |b]. Now, the equation of the circle with center CQ, -2) and r = 218 (@-2)" + + O° = 4. Wnte the equation of the circle which 1s tangent to the x-axis, the y-axis, and the lime 4x + 3y ~6 = Om the fourth quadramt The center is C(a, b) = C(r, -1), sce at as tangent to the axes in the fourth quadrant, The circle 15 tangent to the hne 4x + 3y ~ 6 = 0, and the distance of C(r, -1) to the line is equal to r Wa 61 ay gp = Ir 1-6 = 40 = 1iy=—2) 2 Smce rs non-negative, r = 1 and cq, 1) = Cd, -). We can now write the equation of the circle as (x — 1)" + Gy + IY Eqitisnef Chee 108 = Check Yourself 3 1, Find the equation of the circle which 15 tangent to the y-aas in the third quadrant with radius r = 3, 2. Find the equation of the circle which 1s tangent to both axes m the second quadrant with radius r = 45. 3, Wite the equation of the circle whose center is on the Ime x + 2y ~4 = 0 whichis tangent to both axes m the second quadrant 4. Find the possible equations for a circle which 15 tangent to both axes and passes through PQ, 2). Answers 3.0 + 4) + YAY = 16 4, (5) + Y~ 9)" = 25 OF (- 13)* + (Y— 13)° 169 B. GENERAL EQUATION OF A CIRCLE ‘We have seen that the standard equation of a cizele with center C(a, b) and radius ris a + Wb =F By expanding this equation we obtain xo-2ax + a? + y—2by +b XO 4 yt 2ax —2by + a? + 6-17 = 0.) 2a, E = -2b, and F = a" + b*—r° and substitute m (1), Wetnd | #49? +Dx + Ey +P=0 general equation of a axcle The equation x4 + y* + Dy + Ey + F = Owhere D = ~2a, E = -2b, and F called the general equation of a arcle Now, let us take D = The general equation of a circle gives the following properties, Dae |, So the radius r Vata iyi Arya of Lies ard Ctr Find the center and radius ofthe circle whose equation is a + y? + 6/3 — 12WDy + 18 = 0 EE 16 Solution 1 Solution 2 Equation fe sr Find the general equation of the circle with center C(-2, 3) and radius r= 3, The general equation of the curcle 1s x° + y° + Dx + Ey + F = 0 where D = -2a, E = -2hand F = D=-2-(2)=4,£=-2.3=-6, andF So the equation is x* + y* + 4x ~ Gy ~ 12 ‘The standard equation of the circle 15 (x @)" + (y~b)° = 1°, so @ +9 +O -3°=5 xi bdr 444 y'—6y 49-25 =0 and we getx" + y" + 4x —6y — 12 = 0, ‘The circle x? + y? + 4x + (1 hyy + k-3 = 0 passes through the origmn. Find its radius, The origm O(, 0) is one of the points of the circle, so its coordinates satisfy the equation, OF +04 4-04 -W)-04k-3 =0,k-3 =O. So the equation 1s x° + y° + 4x -2y = 0. Now.r= ADEE —aF 5 r= LYFE H0 5 r= a We could also find the radius by using the center, The cele x+y? + 4x —2y =O has center Cea, b) = c%-2, 2 and the distance between C(-2, 1) and the origin as the radius, so r= (yet = 1G the crete? + yf + 4x + hy + kB = O18 tangent tothe pans. Find k, Solutio Since the circle is tangent to the y-axis, 1t has center C(a, b) such that r = Jal. is 1 re ah By: a +R -43k -8) 1G +R -12k+32=16 HW -12k+32=0 (h-8)(K-4)=0, 80 =8 or ke Check Yourself 4 1, Find the center and the radius of the circle x* + y° + 8x ~ 10y -9 = 0. 2. Witte the general equation of the eirele with center C(-2, 3) which as tangent to the line 3x + dy —16 = 0, 5, Find the radius of the exrcle x* + y* — (6 — myx + my — 3 + m) = 0 which passes through the pomt (-2, -1). 4. Find the length of AB if the circle x* + y? + intersects the axis at A and B ay -12 -3. Find k. 5, The circle x° + y? — 4x + 8y + k = 0 is tangent to the line Answers 14,5); 5% Axes +av-oyt9=0 35 47 9-5 discriminant of a excle Given the equation x + y" + Dx + Ey + F = 0. 4P is called the discrimmant of the cucle the expression D* + | When D* + —4F <0, Band the equation does not represent a circle, When D* + E*~4F = 0, and the equation represents onlyapomt (-2, Fy When D* + B* —4F > 0, re IR, and the equation represents a circle with center at E, ©) and radius — Eu 19 Solu ES 20 Solution Equation of e Cirle In the general equation of a circle, if F < 0, then the discriminant D* + E* ~ 4F > 0, So it always represents a circle, Find the possible values of p such that x* + 3 6x —4y + p = 0 represents a circle. To represent a circle, the discnmimant must be positive D+ E4P>0 6)" + C4)°- 4-p > 0, 80 52-4p > 0 and p < 13, Ix’ + y°— 2x + y—m + 1 =O represents a point, find m fats just a pont, r= DP + ES 4F = 0, 50-2)" + 1 4-@m+1)=0 and 4414 4m—4= 0, Now 4m = -1, som ‘The equation in the form of Ax* + By* + Cxy + Dx + By + F = 0 1s called a second degree equation in two variables x and y ‘This equation represents a circle if 1 A=B20, 2 C=0,and 3 Dye E Qe Note ‘The simplest form of the curcle equation Ax* + By? + Dx + By + F A=B=1 Eu 21 EEE 22 Solution Solution Solution ‘The equation (m ~ I)x* + (2m —4yy° + 4x + 6y +4 represents a circle. Find m. ‘The coefficients of x* and y* must be equal, 1. €. m-1=2m—4 m =3. Now, ler’s check the discrmmant ofthe equation 2 ° AF _ Ay Ge 44 IG A G 9 2y? + 4x + By + Ora =5 >, so itis indeed the equation of the circle Show that the equauon 2x° + 2y" — x + 6y + 8 = 0 represents a circle. Determme its center and radius, ‘The coefficients of x* and y* are equal and the coefficient of xy 18 0, so we can arrange this equation as 2Q? +a? 4e + By + A) = xt yar + 3y+4=0 D+ E-4F = (4) + 3° -4.4=9>0. Since the conditions are satisfied, the equation represent a eirele. Now, a= = Ala ana p= -F =>, 50 0, 3) wine comer LEAF 1 CETTE =F 23 ns Check whether each equation represents a circle or not. If it does, determine the circle’s center and radius, a x +a 12xy +5 =0 bata tae +350 ce t+ yt Bey +7 xt + yh + Qe ay. ° a, There is a term containing xy in the equation of the citele, Sox" + y"— 12xy +5 not a cirele equation. 01s b, This 1s not a curcle equation since A = and B=-1,A#B. ©. This is not a enrcle equation since A = 1 and B= 3,A#B 4. If we compare this equation with the general equation of the circle, then we find D=2,E=—-AmdF=-11 So, D' + £*—4P = 64 > 0, and this equation defines a circle. tits a a et Cicer The coordinates ofthe center are a=—2.= 1 and b a ‘The radius of the exele is. r= AWDF FEF AP = 3 VGT= 4. Thus, the center is C(-1, 2) and the radius 1s r= 4 umts. Alternanvely, we could also find the center and the radzus by rewniting the equation of the carcle as a perfect square. xt ty" + 2x—4y—11=0 4+ 2y 4 yf ay-11=0 M4 Qe 1-1 buhay t4—-4— = 0 @ + +y-O 144411 1 + 2)" = 16 So the center is C1, 2) and the radius is r = 4 units, 24 THe mans ofthe ence 3x° + 34° —3y + 9y~ 3430 ts thee unas. Find m Solution Aer diding the equation by 3.we gets? taf ty Be 1 ml. Le +O" -4-F 4D i r= LT FE Csr But r Check Yourself 5 1, Find the center and radius of each cirele. a.x'+y'= 16 b. Ox? + 9y? 8x + 129-6 = 0 2, Find the possible values off such thatthe equation 2x” + 2y? + Oka + DViy + 2h + 10 = 0 represents a end, 3. The equation (m ~ 3)x" + @ ~ my + mx ~ 6y + m = 0 represents a circle. Find the radius and the center of the e1rcle. 4. The equation 2x° + 2y° - 6x + 8y ~ 4k = 0 represents a pomt. Find k. Answers aeetakes 3029 ;3 4-2 Eqitisnef Chee m = EXO 25 Solu EE 26 tic Arty ‘White the equation of the circle which passes through the three pomts K(-1, 0), L(0, 2), and MCL, -2) The general equation of the circle 18 x" +" + Dv + Ey + F = 0. The coordinates of the points K, L, M must satisfy this equation because they le on this circle for KC-1, 0); 140-D+0+F=0 D+F=1..@) for L@,2);0444+0+2F+F=0 | 4 2®4+F=-4 (2) for K-1,-2);14+4-D-9E+P=0 | -D-2E+F=~5 ..@) Ifwe add (2) and (8) side by side, we get -D + 2F = -9, and if we solve this equation simultaneously with (1), then we obtain F = 8 and D Substituting F = -8 in equation (2), we find E = 2. If we put the values of D, E, and F in the general equation, we find the equation of the curcle is x* + y?- 7x + 2y-8 = 0. Find the center of the exele which circumseribes the mangle formed by the lnes 1 4 x4, y=4x—4, and x Since the product of the slopes of the lines [ y=-SxtDand y=4r-4 1s -1, the mangle a 15 a night Mange. So the center of the circle 15, on the hypotenuse, that is, 15.0n the Ime x = 0 The tine y=—4x +5 mmerseets the y-axis at A(, 8) and the lime y = 4x ~ 4 intersects the yas at BQO, 4), Since AB is the diameter of the cucle, the center is the midpoint of A and B. a: ‘Therefore, the center is C(O, Check Yourself 6 1, Find the equation ofthe cirde which passes through the points A(, 1), B(, 3), and C(-1, 2). 2. Find the equation of the circumscribed eurele of the tangle formed by the lites y = 3x ~3, 3x #11, and the ans 0 2at + yt—34r +33 ia amd Cire EXERCISES 2.1 A, Standard Equation of a Circle L. The equation @ 3)" + (y + 2)° = 118 gen a. Find the center of the circle D. Find its radius. ©. Show the eircle in the coordinate plane. 2, Wnte the equation of the circle whose diameter 18, KT with KCI, 2) and T(6, 6). 3. Find the center and the radius of the circle Stet 2 4, Find the equation of the circle with center CQ, -3) which is tangent (0 the me 4x ~3y + 9 = 0. 5. Find the equation of the circle which passes through P(1, 3) with center C(4, -1), 6. Find the equanion of the cemtral circle which passes through P(-1, 3), 7, Find the equation of the curcle with center C(4, -3) whichis 4. tangent to the x-axis, D. tangent to the y-axis, 8. The cucle (x + m=-28 + y + m-3)' = 258 tangent toboth axes in the second quadrant, Find 9 Find the equation of the cxcle with center 0) and radius r = 3/2. 40, Find the equation of the circle with center C(O, 7) and radius Equation fe Cite LL, Find the equation of the exrele centered at the onigm wath radius r = 10. 1. Find the equation of the circle tangent 10 the x-axis wath center C(5, -2) 13, Find the equation of the earcle tangent to the y-axis with the center C3, 1) 14. Write the equation of the circle tangent to both axes m the fourth quadrant with radius r = 15. 15, White the equation of the circle tangent to both axes in the third quadrant, whose center 3s on the ine x ~5y-4=0 16, Find the radius of the carcle tangent to both axes ‘which passes through the pomt P(I, 3) U7, Write the equation of the circle whose center is ° on the Line x ~ 2y ~ 6 = 0, and which is tangent to both axes. Equation of a Circle 8. Find the center and the radius of the circle 4G. Find the general equanon of the circle with center C(-3, 9) which is tangent to the x-axis, 20. Find the general equation of the circle concentric with the arele x* + y* + 4x + 3y +4 = 0 and tangent 0 the 1-axis ‘The equation (= mx* + 2y° - (m—3)x— 129-6 = 0 represents a circle, Find the center and the radius. of the carce. - = 22, The cucle x? + y° + 4x + — By + k-3 passes through the origin. Find its radius. 23, The equation 3x* + 3y° + 18x + 12y—3k = 0 represents a are with the radius r = 6 units. Find k 24. The equation x* + a point. Find m. 2x + 3y-2m =O represents 25. Find the equation of cirde with the center C1, 1) which passes through the center of the cucle x? + y?— 6x + dy —2 2G. Find the equation of the circle which passes through the pomts AQ, 1), BQ, 0), and C(I, 0). 27. Find the general equation of the circle whose center is on the ne 3x ~y ~6 = 0 and which is tangent (o both axes mt the fourth quadrant, 28, Find the general equation of the circle whose center is on the line y = -1 and which is tangent to thelnes x = 1 and x =7. 29. The equauion Ok Dx" + y' + @ Dyx + xy -3=0 represents a circle, Find ke p. 30, Find m such that the cwrcle x* + y*—2y + m= 0. 45 tangent to the line x = 2. Find &: p Soon 81, For AABC, m(ZBAC) = 90°, BC2, 4), and C(, -4) are gven. Find the equation of the circle which circumseribes SABC. | 32, ABC 1s an equilateral triangle, B(1, 0) and C(7, 0) © are two of the vertices, and the vertex A is mn the first quadrant, Find the equation of the circle inscribed im AABC. 33, The nes xy +4 =0,x +298 =0, andthe © yams form a triangle. Find the equation of the cacle which circumseribes the tangle. 34. Find the equation of the circle which passes through the origin and the x- and y-mtercept of the line 8x + 4y —24 = 0. 36, Find the equation of the carcle whose center 1s on. the line 2x ~ y + 2 = O.and which 1s tangent t0 the bnes x + 1 = 0 andx~3 =0. 36, Find the equation of the circle whose abscissa at the center 1 -2 and which 1s tangent to the parallel lmes 2x + y -3=Oand2x ty +7=0. 37, Find the equation of the circle whose ordinate at the center is 4 and which as tangent to the parallel Imes 3x ~ 1 + 4 = Oand 3x-y~2=0. 38, Find m such that the cade as tangent to the x-axis, Wf + 6x—8y +m=0 39, The line PK with endpomts P(3, -1) and K(a, b) 1s the diameter ofthe euclex* + y° 4x + 6y + F Finda +0 +6. AQ, Find the radius of the cele passing through the pomts AG, 1) and BCI, 3), whose center is on the Ine y =3x-2 iyi Arya of Lies ard Ctr (E> POSITIONS OF LINES AND CIRCLES A. RELATIVE POSITION OF A LINE AND A CIRCLE pomt-of tingeney Ame which meets a circle at only one point is called a tangent to the circle, and this pomt 1s called the pomt of tangency Let C bea circle with the center C(a, b) and radius r, and let (be a line. The distance from the center (0 the Ime [18 CH = d. There are three different possibilities for the position of the me relanve to the cicle. \ "i > 1M ar =r der the line does not intersect the tine is tangent the tine intersects the circle the eile to dhe eee ‘at ovo different pots ne~2) ne = (i) ne = (61) SY 27 deserve the position of the curcle (x -2)" + (y + 3)" = 25 relanve to each line. a. Qe 4y + 12=0 b,x t4y-19=0 ©, Bx—ay +2 Solution C(, -3) andr = 5 are given. We must find the distance between C and the hnes. Sem A 4 [3-244-¢9)-101 342 A tine ih ats, which nego lea so den pois elle sean Be ai dissin 2B Te arte w+ 0-2 x= 2y + k= 0. Find k. Postion of Lins and Cinco 1s = =6>1, so the line does not mtersect the cucle. bd= 1, so the he 1s tangent to the circle. =4 0, then the line is a secant line. It intersects the circle at two distinct points and these pomnts form the solution set of the equation, ESTEE BT escrne the rtatve poston of y = 1x and.x' + y!— Gx +7 =. Ifthe tine and ewcle ae tangent, find the point of tangency. Solution Let us substtute y = 1 — x an the ele equation, (1-n*-6r+7=0 ; 2x8 48=0 4x44 A = bY —4ac, 50.4 = 16 - 4-1-4 = 0, and so the hneis tangent to the arcle, The coordmates of the pomt of tangency are gwen by x" — 4x + 4 = 0, $0 @e 2)" and x ‘The solution of x*- 4 + 4 faves us the abscissa of the mtersection point ‘To find the ordinate, substitute this value in the line equation y = 1 — y = 1-25 =1, so, ~1) 1s the pomt of tangency. Posbinns of Lins and Cis 7 = BQ find au posste values of kif te nex — y + k = 0s tangent tothe cree x? + y Solution We replace y = x + kam the equation x° + wre a Det RE If the Line 1s tangent to the circle, then A must be equal to 0. Thus, 32 kaaayi 4,50 5 2a" + hy + I-40, ANE 4-2-4) = 0; AWB +320; 4K EEEMTITY BB fra tne coordinates of the mrerseetion pomts of the he x -y + 1 = O and the eircle xe ty?-3x-y-1 Solution Subsmtute y = x + 11m the circle equaton x4 (+ D'-3r-r + =I tat tnt 1—3e—x-1- A= Db -4ac = 4-4-2. (1) cpad® | 248 1 a xt we get WEEDON 34 wa + 2 +p = 0s tangent tothe rans, Find p. Soluti ‘The cele is tangent to the y-axis, so let us substitute x = 0 in the equation, We get y°—y + p = 0, and we know A = 0. Di 4ac = 0 1-4-p=0 zt 4 ne Analytic Analysis of Lines and Circler Check Yourself 8 1, Examine the relauve position of y = —2x + 1 and x* + y+ 2x + 2y— 11 = 0 and ond the coordmates of the intersection point(s), 1f they exist. Sx 4 4y + 6 = 0. Find the length of AB 2. Aand B are the x-imtercepts ofthe circle x° + 3. Find r ifthe line 2x ~y ~ 4 = 0 is tangent to the circle x° + 9 4. Find the length of the segment of the line y = x + 1 which hes inside the circle 2x" + 29? + 4x — By - 10 = 0. 5. The Le y = x 1 does not intersect the circle x" + y + 2x -3y + k- 1 = 0. Find the possible values of I. Answers 4 1. 2, 5) and ( 3 Son aE aa one a 5 8 B. EQUATIONS OF TANGENT AND NORMAL LINES Postions of Lins and Cirle normal fine A line which 3s perpendicular to the tangent of a La cucle at the point of tangency is called a normal lune of the circle. In the figure, the line t is tangent to the circle at (vy Y,) and the Line m 15 normal to the circle at PCy Ho) 1. Equation of the Tangent and Equation of the Normal Through a PointonaCircle =, ———~& Let PC, ¥,) be a pomt on the ettele @-aP + y—by srt 148 the tangent passing through P(x, 1.) and 1 15, the normal to the cxtele at pomt P (t Lm). ‘The nonmal passes through the point P(x,, y.) and the center C(a, b) we so the slope of the normal is | m, = | ‘The tangent ine 1s perpendicular to the normal lime, so the product of their slopes is equal to —1 | =a mom, = tvand so m= 2, (my =F a. Equation of the tangent through a point P(x, y.) Te spe ofthe agents m =F and Pon) a pat ofthe angent sobs Yong ne tha otic p i th pnt yy, = ei, tan oi hut oh tangent as zz “Ge to) jor yb | ©) Ge-b) + 4) G—@ = 0 b. Equation of the normal through a point P(x. ¥.) The dope ofthe nomnal isp, —2= and Py x9 is point of the nom, soby usm he si ago so a BBB vind thn cancion tioeormalend fielameent tothe ciel be A teil —10 ot: the point P(-4, -1). Solution 1 The center of the circle is C(-3, 2) |. ‘The dope of the normal is m, = 22—# = and the equation of the normal 1s yayo= mx) py L=3-@ 44 sy — Solution 2. P(y 4) ES 37 Postions of Lins and Cirle 2, The slope of the tangent is m, So the equation ofthe tangentis—-y —g, =m, @—m) ytia-t w+a xtayt7=0. P(A, -1) and C@, b) = C3, 2) 1. The equation of the tangent is Y=) Yo~b) + =X) Oy ~ a) CD) 1-9 F&F D-CA F3)=0 “By-3-x-4=0 5 x43y47=0 2. The equation of the normal is (Y= Yd)» Bp a) ~ =) Yo W-CD)- (A+ 3)-@- C4) C-2) -y-1 43 + 12=0,503x-y + 11=0. Find the equations of the tangent and the normal to the circle (x + 1)? + (y -2)* = Sat the pomt PCA, 3) ‘The equation of the tangent passing through P(1, 3) 15 (a) a) +B) Y— 4) (+ 1)-@-1) + B-2)-G-3)=0 2.@-) +1-G-3) ax ty-5 The equation of the normal passing through P(1, 3) 18, Wy —b) —%) ~ Oy - a) YY.) = 0 @-%-@-)-A+)-G-3) w-1-2y +6 $0.8 2y +9 Find the equations of the tangent and the normal to the circle 3x" + 3y° + (m ~2)xy — 3x + 3my ~ 63 = 0 passing through the pomt P(-2, 3). ‘The coefficient of xy must be zero, so m — 2 = 0 and so m = 2. Therefore, the equation is, 3x" + 3y* 3x + 6y 63 = 0. Dividing the equation by 3 gves x" + y°—x + 2y—21 = 0. 50 C(a, b) = C1 c=) te can = = The equation of the tangent 1s o-9- 4 DF eH 2-4 ‘The equation of the normal is (y - 3) - (2 (42) G41) = 0.46.5 48x41 Check Yourself 9 1, Find the equation of the tangent and normal Imes to the circle (« -2)* + @ + 3)" = 13 Which pass through the point KS, -1) 2, Find the equation of the tangent and the normal to the circle (x — 1)" + @ + 2)" = 20 ‘which pass through the point P(3, 2). 3. White the equations of the tangent and the normal to the circle x* + y* — 4x + Gy - 12 = 0 at the point P(S, 1), Answers 1 3x +2y-13=0 2 ext wy-7=0 St 3x + dy 19 n: 2x —3y- 13 =0 mde y-4 n: 4x —3y 17 =0 We can simply find the equation of the normal and the tangent to the central exrele tang x? + y= Pata point on the circle. the slope of the slope of the normal is, the tangent 18 % ‘The equation of the tangent 1s therefore Bein i ey = gn eat yet tR aE HY YY, eX, ‘The equanon of the normal 1s therefore Beem) | Ue 3 to Xe ys Mo Wed, Analytic Analysis of Lines and Circler Solution Postions of Lins and Cirle Find the equations of the tangent and the normal to the circle x’ + y° = 10 at the pomt PG, D. ‘The equation of the tangent passing through P(3, 1) 1s yoltx-3=10 ; 3x+y-10=0. ‘The equation of the normal 1s, Yo Yy HX YeX— Xe yo= 0 ¢ yoB-x-1=0 5 3y-x c. Practical ways to find the equation of the tangent through P(x. ¥.) 1, The equanon 2° + y? * can be wnitten asx x Fy yy P(g a) 18. pomt on the cicle and when we substitute the coordinates of P(t, J.) Mt0 the equation of circle, we get the equation of the tangent line: x- x, + yy = 1! 2, The equation (x ~ a)" + (y—b)* =F can be wntten as (a) -@-a) + Y-b)-Y-By= FF So the equation of the tangent Line Urough P(t4),) 18 (=a) (=a) + (Y~b)- Gyo) 8, The equation x° + y' + Dx + By + F = 0 canbe written as D E xextyyt2.@tyt Suter puts @tEDt SO tD So the equation of the tangent line through P(X) 8 D E ext yg t 2 txt 2 Wty + F=0. tN Het HDT OIE Write the equation of the tangent drawn from A@, 3) which is on the circle x+y + 4v-By +3 =0. ‘The equation of the tangent at the pomt (x,, y,) to the encle x + y° +4 ~8y +3 = 015 D E mihefomr-r ty ut 5 He) +S Uta) +P =0. ‘Thus, 4 * Wet > AtHN- 7 Bty+s Qe + 3y 4442-124 +3 = and so 4x — y—5 = 038 the equation of the tangent. = = Check Yourself 10 1. Find the equauons of the tangent and the normal to the curcle x* + y PCV, 25), 2. Find the equation of the tangent to the circle (x ~ 6)" + (y + 4)" = 9 at the pont P(6, ~1), 3, Find the equation of the tangent to the circle x* + y°—2x + 8y + 9 = Oat the pomt (3, -2) 25 at the pomt Answers 1 tQysxt SVB Qy=-t Sx ty-1 x ney 2. Equations of Tangents Drawn from an External Point There are (vo tangent lines drawn from an external point of a circle. Let y = mx +n be the equation of a tangent Line. 1, PG, y.) satisfies the equation of the tangent Ime y= mx +n. 2. We can use the condition of tangency of a Ime to a circle and the distance formula ‘between a point and a line to find the values of m and n, When we substitute the values of m and n in the equation y = mx + n, we get the equanons of the tangent lines: (EES 4G ine the equations of the Lines tangent to the cixcle (x - 2)" + (y ~ 3)" = 4 drawn from the external point P(6, 3). Solution Let y = mx + n be the equation of the tangent tne. (6, 3) is on the tangent line, so 3= 6m +n,1e.n=3-6m ‘The distance of C@®, 3) to the tangent Ime 26,3 ‘This qves 2ym*F1=|2m -3+ 0] Ex 41 Solution Postions of Lins and Cirle When » = 3 ~ 61 1s substituted in the equation, we get dm +1 = [2m —3 +3—6m|, 1 Wm* +1 = |-Am| or 4¢m* + 1) = Cam)" 4m? +4=16m" 5 12 Since n = 3~ 6m, for m, and m, Hence, the equations of the tangent lines are x+3-248 8 7s teiy= ant 34 28, ‘Wnite the equation of the line passing through the pant K(S, 0) which as tangent to the cde x' + y' = 16. the frst quadrant, Ais the point of tangency: 1 = mx + nos the equation of the tangent. KG, 0) = K(x, y,) and y= mx +n O=9m+n n= —3m A) 5.0) OA = 4 and C(a, b) = CO, 0) 50 [tO ve f= avin 4 Jie m 16(1 + m*)...2) Solving (1) and (2) simultaneously, we get (omy = 16 + 16m? ; 9m? = 16 ; m=44 na n=z 2 3 3 The equation of te agents there y= $22 and Since the tangents in the first quadrant, S.r+ 2 ts the equation of the tangent = = Check Yourself 11 10 drawn from the 1, Find the equations of the lines tangent to the exrcle x° + extemal pomt P(2, 4), 2. m, and m, are the respective slopes of the Imes tangent to the circle (x —2)* + (y 1) drawn from the external point P@, 4), Find m, + 1 y 3. Im the figure, d is tangent to the circle at T, and the cucle is tangent to the xaxis at the ongin. The equation of the hne is 4x ~ 3y — 12 = 0. Find the equation of the circle. Answers 1 y=-3r410;3y=x4+10 2-2 C. RELATIVE POSITION OF TWO CIRCLES 126 Let C, and C; be the centers of two circles, and let r, and r, be the radii Then CC, = dis the distance between the centers of the circles. L If d > 1,4 ry the circles do not intersect. Id =r, + r, the extcles are tangent t0 each other externally. B.A [ny r, and we get d’~ 1 = P > 0. =P=0. > If Qc, y,) les on the cucle, then d = r, and we get d’ 5. IfQ(x,, y,) lies in the interior region of the circle, then d 0, so QG, 3) is im the exterior region of the circle. Also, QT as the length of the tangent segment and OT = ¥P, so QT = 3 untts Conclusion 1. The power of the point Q¢x,, u,) with respect to the circle Se ytertPaatey 2. The power of the pont Q(x, y,) with respect to the excle (a) + Y— by =F is P= a) +, bY 3, The power of the point Q(x,, y,) with respect to the carcle xt yh + De + By +P V+ Dx, + Ey, +P. SENT 48 rina ine power of the pomt Q¢-3, 1) wth respeet to the aarcle x* + y" Solution By substituting the coordinates of the pomt Q¢-3, 1) in the equation Q P=(ayer-12 P= “4 yi 1", we get 2 < 0. So the pomt Q(-3, 1) 181m the interior regon of the encle SIA 49 sina the power of he pomt a(S, -4) with respect to the enele (x ~3)* + W +4)" =4 Solution P= (x,—a)* + @,-by-F P=@-9°4 C4444 Pa4-4 P = 0. So the point A, ~4) is on the circle [= 50 Solution Solution En 52 Solution Find the length of the tangent segment dravm ffom the pomt BQ, 2) to the circle xt y + ox + 3y-7=0. Let BT be the tangent segment dravm from the point B(, 2) tothe ctcle. Recall that B7 = v/P and Let us fist fnd the power of the point B(1, 2) with respect to the circle P=I42+5-143-2-7 Pait4atot6-7 P=9. ‘Therefore, BT = vP Br= 9 BT = 3 ums. The length of the tangent segment drawn from the point K(4, ~1) to the circle xi + y" + 4v— Gy + k= 0 1s five units, Find k. LET 15 the tangent pomt, KT the tangent segment Hence, KT = /P vP and P = 25. Pexityt + Dx, + Ey, +P a= 4+ CY + 4-4-6. CH) +k 25 =39 +k h=-14 The center of the circle (x ~ a)" + y x + (y-3)" = 25, Banda 438 in the mtentor region of the circle The center of the earcle (x — a)" + 9" asC(@, 0). ‘The power of the point C(a, 0) with respect to the circle x° + (y ~ 3) negative because 1 18 in the mterior region of the circle, Therefore, P<0 @ + @-3)°-25 <0 a°-16< 0 andweget 4 (2,2) SND TE AP = £150 = 24 units ity? t6r=0 C4(-3,0) = AGF = 3 unis 2. Now, let us find the equation of the radical axis: at y?-4x—4y = 0 xi + yh + 6v=0 =10x —4y = 0 and so 4 ays 1 and therefore, the radical axas is perpendicular to the line joumng the centers, (EQUI 57 the radical ans ofthe cuctes x? + y? + 4x +24 +3-= O and x+y + 3x (m— Dy + m + 1 = 0 passes through the point AC, 2). Find m. Soluti n sity te + 2y + 3=0 x+y + av by +m + 1=0 x+ Qt m-Dy +3-m-1=0 x +m + Dy +2-m=0 ‘The pome ACL, 2) satisfies the line x + (m + Iy + 2-m = 0, so we get 14(m41).242-m 142m 4242—m — atm=0 ; m=-5, Check Yourself 15 |. Find the equation of the radieal axis of the exzcles, a4 af Be + Ay 11 = Oand x? + + Te -9 2, Find the x- and y-mntercept of the radial axis of the carcles Ox" + 2" + dv —2y + 2=Oand x" +y" + 6x + dy + 9=0. Answers 1 1ax-4y +2 2 (2,0) and @,-§) 3 C, RADICAL CENTER OF THREE CIRCLES En 58 Soluti it, Radical Aris, and Rahal Coser radieal-eenter Let C,, Cy, and C, be three noncollmear pomts and the centers of three circles. The pomt of antersecuion of the respective radical axes of each pair of curcles selected from the three carcles is called the radical center of the circles The radieal center 1s the unique point that has the same power wath respect to all three circles. In the figure, K 1s the radical center of the cir- cles with centers C,, C,, and C, To find the coordinates of the point K it is enough to find the mterseetion pomt of any pair of radical axes selected from d, dy, and dy, Find the radical center of the circles x° + y+. x-2y-2=0, x" 4 y+ 3v4y—4 andx + y"-x + 2y-6=0. Furst let us find the radieal axes d, dy, and dy x tyltx-Wy-2=0 P+ x-2y-2=0 xt y+ Br ty-4=0 fx + 2y-6=0 -2x-3y +250 2x-4y +450 dy 2x + 3y-2=0 dx -2y +2 Since d,,d,, and d, mtersect at one pont, it 1s enough to find the Intersection pomnt of any ‘alr, Let us choose the bnes d, and d.. Qe + 3y-2=0 xa 2y +2, Qe + 3y-2 Ox + 4y—4 Ty-6 Ax-y+2=0 -2.42=0. So the pomt 1s on the third axis, t00. Check Yourself 16 1, Find the radical cemter of the curcles (x -1)' + (y + 2)" = 26,2" + y* = 16, and, x+y +5r-6=0, Answers 1 1 ap ‘See, the buffalos have learned to defend themselves using circular ) methods! It'll be quite hard for us to eat fresh meat from now on. Analytic Alyse of Lines and Ctler EXERCISES 2.3 A. Power of a Point |. Find the power of the point Q¢-2, -3) with respect to the circle x" + y"—3x + 4y +10 = 0. 2. The point B(2, 5) 18 at the exterior region of the ence x" + y°—2x + 3y + m = 0. Find m. 3, Prove that the point K(4, ~4) 38 m the interior region of the curcle x? + yx + 3y—17=0 4, Find the length of the tangent segment drawn from the pot A(J, ~4) to the exrcle wt yor t ay +8 5. The length of the tangent segment drawn from the pomt Q¢-2, -3) to the circle (+ 2)° + (y= 1° as three units. Find the radius of the eucle. @, Find the distance from the nearest point on the emcle (x ~ 2)" + @ + N° = 9 to the pomt PCL, 3). Power of «Port, Radical Ari, al Radical Center ‘7, Find the distance from the furthest pomt on the carcle x° 4 y! — 16x ~ 16y + 92 = 0 to the point PG.) 8. The Length of the tangent segment drawn from the point K(3, a) to the curcle x* + 9° = 161s /42, Find a ®, The distances from the nearest and the furthest points on the circle x* + y® 4x + 2y-4 = 010 the point P(3, -2) are m and n, respectively. Find, m 10. mn the figure, Q, A, and B are collinear pomts suich that 5 QA =3-08 ‘The equation of the circle ab ty" 4 3v—4y-29 = 018, aven, Find the length of the line segment AB. LL, ind the coordinates of the nearest and the furthest points on the circle (x -3)" + (y +4)" ongn. tothe = = B. Radical Axis of Two Circles 12, Wnte the equation of the radical axis of each pair of cucles. a @ 1 + +9) = 4and @ +o +UFD =9 w+ +2" 13. Fand te equation ofthe me contaumng the common. chord of the eitcles x° + yf —4x + 2y +4=0 and x? + y®-2x + 6y +5 =0. 14, The common chord of the circles @-1' + Gt 2s 4 and x + y'— 4x + 2y + m = 0 passes through the point A(1, ~2). Find m, 15. The slope of the line passing through the intersection points of tie circles x? + y°— 3x + Sy 6 = Oand 2x" + 2y” + Gx + Jay —4 = 01s 2. Find a. 16. Find the x- and y-itereept of the radical axis of the circles (x + 3)" + (y~2)° = 8 and x+y? + 8y +2=0 C. Radical Center of Three Circles U7, Find the radical center of the circles given by the equations x° + y* = 2, x° + y*— 4x6 = 0, and x4? + 6-8 a — 18, Fmd the coordmates of the radical center of the curcles given by the equations x ty? 4x —4y-1= 0, Dat + Dy? + 4v — By +2 = 0, and 8x° + By" + 18x — 6y - 18 = 0. 19, Find the sum of the coordmates of the radical center of the citcles given by the equations wt y'-4=0, Ot +3 + @-1P Rieti macioue 20. Find the position of the pomts A(-3, -4), BC3, 3), and CQ, -2), relative to the eircle xf ty" =3y-28 21, The radical aus of the cacles xtty' tax +3y-1=0 and 2x + Dy! + Bx + by ~ 9 =018 Tx Sy -3 = 0, Rind ba 22, What are the coordinates of the furthest point on the circle (x ~2)" + (y + 3)° = 128 to the pome AQ, -2)? 23, The radical axis of the circles O ey tx-y-3=0and Bx" + Sy + 1e- 12y 4a Find a asthe bne y Analytic Analysis of Lines end Circle ih 4 FURTHER APPLICATIONS ON CIRCLES (OPTIONAL) A. PARAMETRIC EQUATIONS OF A CIRCLE Up to now, we have defined a circle by a single equation involving the two variables x and y. In this section, we mtroduce a third variable, 0, to determine a circle as a pair of equations, where O1s the positively directed angle between the radius of the circle at a point P(x, y) and. the positve direction of the x-axis. Let us look at the parametne equabons for different exeles. 1. Paramet Equations of a Central Circle Let PG, y) be any pomt on the carcle x” + y" For 0 < 0 2a, 61s the melination of OP In the nght trangle OPH, sin = PH sn = 4 soy = rin. cos 0 = OH; cos.6.=* sox = r008 8. X= reos @and y = 1sin Bare called the parametric equations of the central circle in terms of the parameter 8. SET 5D wate tne parameme equations of the central circle with radius 3 umts, eile x mrcene'd ond.g 3 ole Ul wail = 8 an Gegemmenieoegmsencsnme = ed ond a= tein, Porinstance, when @= 30°, x=3e0s 30°= 2% and y=ssmn 30°= 3, v5 3 so we get the exact position of the point (28°, 3) on the circle. eel P point (“S~, 3) We can see that using different values of 6 will give different points on the circle. Conversely. every point on the circle corresponds to a particular value of @, and so the parametric equations define the circle completely. Further Applications on Cinta 143 = Ex 60 Solution EES 61 Solution Parametric Equations of a Circle @-a)’ + (y-b)y For any point P(x, y) on the circle centered at Cea, by, x = OB + EF =a + reos 8, and y = OL+ LK = b +150, where 0 < 6 < 2r. Pa,» Therefore, the parametnic equations of the carele (x ~ a)" + (y —by* = rare @ + reos 8 andy =b + rsin 6 ‘Wnte the parametme equations of the circle centered at C(4, -3) with radius 5 ums CA, 3), 7 = 5 untts, and 05 6 < 2m, By substituting the given values in the parametric equations x = a + rcos @and y = we get x = 4 + Scos @ and y curcle. + rsin 8, 3+ Ssin 6, These are the parametric equations of the Find the parametne equations of the cucle a‘ + y° + 4x —2y—4 Let us find the center and the radius of the ctele. y= 002, » ETT ; r= 2.634 2 1 2 Therefore, for 0< 6<2n, X= -2 + cos O and y = 1 + 3s 0 are the parametric equations of the given circle. Analytic Analysis of Lines and Circler EXTEN G2 rove sna tne parsmetrc equations x = 5 + 2eos Gand y = 3 + 2sin 8 representa ciclo Solution x = 5 +2cos 0 andcos 6 y= 34 2sin @ and sin @= Taking the squares of both sides of the equations, sy" a G3 H-3)" 4 This 18 a circle equation, Le. t represents a circle. _-3 wwe get cos? 6= After adding side by side, we get and (x - 5) + W-3 = 4 Check Yourself 17 L. Find the parametie equations of the circle x* + 3} 2. Find the parametric equations of the circle x* + y*- 8x + 6y + 16 = 0. 5, Find the equauon ofthe eucle with parametne equations x = -3 + 4cos@ and y where 0 < @< Or Answers Lv=Pewo 2 x=4 43080 3.43 +G-D + 4sn0 16 y= sme y= 3 +350 B. EQUATION OF A SEMI-CIRCLE Let @ - a)" + (y~ be = 1 be a given carcle equation, Ifwe solve the equation for x wy -by. we get (w— a)" ‘The resultant equations are the equations of the semi-cireles separated by the line x = a. As shown an the figure, x=a-yF—@-b) is the equation of the semterele to the left of the line x = a x=atJP—@—by 3S the equauon of the semi-circle the right of the lne x = a Father Applications on Cinta EE 63 ES 64 Solution Solu Let us use a similar method to find the equations | of two semi-circles on the honzontal. | By solving the equation (x a)" + (y—b)" we get (yb) = 1-0, y=bedF—@—ay The resultant equations are the equations of the semi-cucles separated by the line y = b, ‘As shown in the figure, y=b-F— =a 1s the equation of the semi-cirele below the hne y = b. L tory, y=b+ J —G—a)" Is the equanon of the semi-circle above the hme y = Draw the graph of the semierrele whose equation is y= 454-4 Let us rearrange the given equation. y= 64-8 y= 64-3" x'+ y= 64, and the resultant equation represents a.curele with the center C(0, 0) and radius r = 8. Since y= ¥4—¥ > 0 , the equanon y= VOIX is above the line y = 0 (the x-axis) ‘Therefore, the semi-circle 1s as shown in the figure. 8| y= Draw the graph of the semi-circle given by the equauon x=1- JO-(-2)" and Ond the length of the are of the sem-cucle. Let us rearrange the equation, x-1= -P-@-2" (=D =9-G-2" ont +y-2 Since —J pw -2)" <0, the equanon x =1- J6-(y—2)* 1s the equation of the sem-circle to the left of the Line wel ‘The length of the are of the semi-curcle is nr = 3m, Analytic Analysis of Lines and Circler Check Yourself 18 1. Draw the graph of the sembencle given by the equation + (1G Whats the length of the are of the semi-circle? 2 Draw the g@aph of the sem-cuele gven by the equation y= f72—(y-B" . Whats the length of the are of the semi-circle? y Answers Lan 2 6/3x C,. INEQUALITIES INVOLVING CIRCLES For the equation of a cucle given as (x —a)" + (y ~b)* |. The mequality (x - a)" + (y -)* Fxepresents the exterior region of the circle. SME G5 snow me pomts satistvng the mequality x"+ 25 1m the coordmate plane Solution The appropriate regon is the circle stself plus the anterior region of the carcle. Notice that the center of the circle is at 0(0, 0) and the radius is 5 units ‘Therefore, the pomts are in the shaded part of the figure. GG drow che graph of me mequatny Ge=3)* + G4)" > 4 Solutio ( - 3)" + (yA) > 4 represents the exterior region of the carcle with center (3, 4) and radius 2 units. ‘The graph is shown mn the figure, Father Applications on Cinta [EDIE 67 draw ane graph of 0 < 28 + 238, Solution The given inequality 9 9 and x* + y< 36. The solution set of the system is the intersection of the solution sets of these mequalines. yf > 9 represents the exterior region of the circle with the center 0(0, 0) and the radius 3 ums. x’ + y $36 represents the interior region and the circumference of the circle with center O(0, 0) and radius 6 units, The graph 1s shown in the figure [SNE GG show she poits sn the coordinate plane which satisfy oa the system of mequalites. x+Q-Ds4 xed 10, 1 Solution + (| —1) <4 represents the interior region and circumference of the cirele with center C(0, 1) and ot * radius r 2 units x2 1 defines the right-hand side of the hme x = 1 and the Lme itself, The resulting solution is the mtersection of these regions, as shown in the figure GY Wate the appropriate inequalities for the shaded region given in the figure. Solution The equation ofthe einee with the center C(S, 3) and radius r = 3 units as (x -3)" + W -3)° = 9. The equation ofthe ne das Bye 1a We ean represent the shaded region as the system of nequatines 0. and x~y~ (-9y + Y-3)' £9 x-y-150 148 Analytic Analysis of Lines and Circler EEE 7 draw the raph of the system of inequalities ven by ety 0 Solution x + y° < 16 represents the terior region of the circle with center 0(0, 0) and radius r= 4 unt, xy > 0 represents the first and third quadrants of the coordinate plane because the signs of x and y must be the same. Therefore, the graph 1s the shaded region in the figure. Check Yourself 19 1. Show the points sausfving the mequality (@ ~ 27° + @y + 1)” = 9am the coordinate plane. 2. Draw the graph of the system of mequahities 1 < (x + 2)" + 9° <4. 3. Draw the graph of the system of mequaliues (x + 1)’ + (y- 1)" >9 | ysi-x] D. FAMILY OF CIRCLES fumly of curcles The set of all curcles passing through the intersection pots of two circles 1s called a family of etrcles. Let A and B be the intersection points of the two circles C, and C, given by the equations Dex + Bay + Fy = 0. Cuxt y+ Dx + Ey + F,=Oand Cy x" + Then the equation of the family of circles passing through the pomts A and B is ty + Dx t By +P, + h(x’ + y+ Dex t By t BY =0,ke R. ic, +k-C, Father Applications on Cinta

You might also like